Sie sind auf Seite 1von 61
MICROBIOLOGY TOPNOTCH MEDICAL BOARD PREP MICROBIOLOGY HANDOUT BY PACIFICO ERIC E. CALDERON, M.D. For inquiries, visit www.copnotchboardprep.com.ph or https://www.facebook.com/topnetchmedicalboardprep/ Roadmay E TORE PAGE Buster Structre Generales ——1—| Tih, Bacterlal Growth Cycle Infecdon Belo 2 IV. Bacterial Oxygen Metabolism Diagnoate Microb o W Bacteria Corea [Boctericlogy 12 | VI. Normal Flora Mycology ay fy Virsiogy 36 ((Parantorogy #7 Co oveRvEW sd MUST-KNOW TOPICS TROMARVOTES 1 Dilference amorg prokaryotes and eukaryotes 1 Birerences tong bacteria vrubes, fungi and paras Ree Ce {Important normal fora SUS i + Major mechanisms of pathogenicity + Laboratory diagnosis WHAT ARE MICROORGANISMS? ‘© Minute organisms + Too small to be seen by the unaided eye * Individually WHY STUDY MICROBIOLOGY? * Cousative agents of infectious diseases + Normal flora + Environmental importance ‘0 Decomposers © Produce oxygen (© Food chain co Sewage treat, ‘+ Industrial importa © Food industry co Brewing industry © Pharmaceutical industry ‘0 Genetic engineering + Research > genetics and metabolism 0 Simple cell structure © Rapid rate of growth ‘Inexpensive to culture MICROBIAL GROUPS + Microorganisms ‘Bacteria, protozoa, microscopic algae and fungi + Aceliular infectious agents © Viruses, viroids and prions ‘Mobile genetic elements ‘0 Bacteriophages, plasmids and transposons + Animal parasites ‘o Nematodes, rematodes and cestodes TOPNOTCH MEDICAL. BOARD PREP MICROSIOLOGY HANDOUT BY PACIFICO ERIC E, CALDERON, M.D. [DNA within w nuclear membrane ne ver itote dvision No Yes DNA etsociated with stones Ne. ves [Chromosome number ‘One _| Mare than one [Mambrane-bound organelles No ve sae f ribosome Fe [cellwall containing peptidogivean | Yes No Prokaryotic ribosomes: 50+ 30 = 70 ‘stands for svedberg unit, os enon ‘SVEDBERG UNIT ‘© Non-S1 unit for sedimentation rate «Sedimentation rates the rate at which particles ofa given size and shape travel tothe bottom ofthe tube under centrifugal force + This reflects the rate at which 2 molecule sediments under the centrifugal force ofa centrifuge «+ The svedberg is technically a measure of time, and is defined as exactly 1095 (100 f5) EUKARYOTES nt ET etiam memes emer Page 1 of 71 For inmulelen visit www, toonotchboardaren.cern.ch or httus://www.facebook.com/topnotchmedicalboardprep! Fed sedpieseingre-seninn opty rulittien: Ainness-~imeninigendilfin hoe Rollo ed-weelAeabiatd URS For inquiries, visit www:topnotchboardprep.com,ph or https://www.facabook.com/topnotchmedicalboardprep/ ee Teas ea Foi = = Approximate diameter (un)* |0.02-0.2. hes — |3~10 (yeasts) 115-25 (trophozowes), Nudeic aod [Either ONA or RNA [Both DNA and RNA JB007 DNA and RNA [Both Ona and ANA ‘Type of nuceus None Prokaryetic |Eukarvotic |Evkaryotic. —— Taveene [708 [0s [eos aan ocho [absent [abet [Present [present 2 Nature of euter surface [rote capsid and ipoproten enveone| Rigid wal coneaining pepBdoqivean Rigid wal containing chin |Fiexibla membrane a cae Tome Rene ro Samaras ~~ fos ror Sx Saami OU es ee prokaryotas fom eukaryotes? ‘A Relatively smal size and absence ofa nuclear membrane. | nuclear area Which ofthe folowing organisms lack membrane sterol? A. Yeasts 3. Mycoplasmas | €: Protonoe . Staphylocoeet |Win a true about the growth af trmally lmorptie ng?) ‘A. Molds on artificial culture medium at room temperature, | yeasts In iniected tissue 8. Yeasts on artificial culture medium at room temperaturo, | molds ninfecte issue 6. Molds an yeasts on artifical culture medium at room temperature _Moldsand yeasts in infected tue i vinoiws * Obligate intracellular parasites of plants + Acellular ‘Naked RNA ‘+ Nohuman diseases known ‘TRANSPOSONS -> “transposable” elements + Mobile genetic elements : + DNA pieces that move readily from one site ta another either within or between the DNA of bacteria;plasmids, and bacteriophages. > . nw reaisiance raneposase” gone ene Rn Raprussor ia a ‘en0 + Code for drug-resistant enaymes, toxins, or metabolic enzymes + Cause mutations in genes into which they insertor alter the ‘expression ofmearby genes. 4Viruses PRIONS * Noneellular infectious proteins Naked proteins that have th¥'same amino acid sequence ‘certain normal human eal surface proteins but have folded differently ¢ «* Resistant to nucleases, proteases, many chemicals, and normal autoclaving «* Assoriated with spongiform encephalopathles, eg. Creutzfeldt- Jakgb disease, kuru, faalfarlial insomnia () Terdary structure ofthe normal (PrP) protein as determined by NMRspectroscopy (8) proposed model ofthe abnormal, Infectious (PrPSe) pion protein, which consist largely of beta-sheet The actual tera Bruture ofthe prion protein has nt been determined. The two tmoleeles shown inthis igure are formed by polypeptide chains that can ‘be Identical in amino acid sequence ut fld very diferenly As aresult of the differences in folding, PFPC reman soluble, whereas PrPSc produces aggregates that kil he coll (The two molecules shown n this gure are ‘alld covformers because they differ only In conformation) Trends Biochem Se21:483, 1996. DYSFUNCTION DUB TO PROTEIN MISFOLDING PATHOGENESIS OF PRION INFECTION Omran 0 1Y PACIFICO ERIC E. CALDERON, M.D. Orem Page 2 of 71 PREP MICROBIOLOGY HANDOUT 3 TOPNOTIH MEDICAL BOARD herent : wvew.facebook.com/topnotchmedicalboardprep/ For inquiries, visit www.topnotchboardprep.com.ph or https: © Fes rerverer nen: sono ony wcronocy asaout ay serio Ene cADERON, 2, For inquiries, visit wvrw.topnotchboardprep.com.ph or https://www.facebook.com/topnotchmedicalboardprep/ ‘SPONGIFORM ENCEPHALOPATHIES * appearance of the vacuolated neurons with loss of function and the lack of an immune response oF inflammation HUMAN PRION DISEASES disease a + Variant CID (vC}D) + Gerstmann-Strdussler-Scheinker (GSS syndrome) + Fatal falia insomnia (FFI) «Sporadic fatal insomnia ‘ANIMAL PRION DISEASES « Seraple (sheep and goats) + Transmissible mink encephalopathy * Bovine spongiform encephalopathy (BSE) > mad cow disease « Chronke wasting disease (mule, deer, elk) ‘TRANSMISSION via infected tiasue ‘through cuts in skin ‘transplantation of contaminated tissues (cornea) ‘Ouse of contaminated madical devices (brain electrodes) congestion of infected tissue (cannibalism) Via inherited syndrome ‘+ May be sporadic SUSCEPTIBLE POPULATION * women and ehildren ofthe Fore tribe in New Guinea # neurosurgeons and brain surgery patients «transplant surgeons and transplant patients ‘SPECTRUM OF DISEASE * progressive, neurodegenerative disease loss of muscle control ‘shivering ‘omyocionic Jerks and tremors ‘oles of coordination rapidly progressive dementia death xs ‘TREATMENT and PREVENTION a + no treatment available rc «cessation of ritual cannibalisri ‘elimination of animal products from livestock fep «disinfection of neurosurgical tools and electrod (05% hypochiorite solution or 1.0 M sodi ide or autoclaved at 15 psi for 1 hour Prions are not normal and maj ‘exhibita problem with which of 8 | A. Felding of segments ofpolygeptide into geometrically 1 Sederet unit : 8B, Amino acid sequeiéeig a palypeptie chain : Protein assembly into the mature polypeptide and its |” componenttagais . Nambetgrd types of plypeptde unks of ogomerle proteins dhathelr pail arrangement PrP knockout mice, which are engineered not to express the | PrP gene, show no obvious pathological phenotype. However, | these mice have been shown to have abnormalities in all ofthe I BACTERIAL STRUCTURE “GACTERIALSHAPE ARG SIZE ‘three shapes: ‘ococe! (spheres) ‘bacilli (rods) o splrochetes (spirals + cocel arranged in three pattern: ‘pairs ip chains (s ‘clusters (staphylococe) BACTERIAL CELL WALL « All bagteria have a cell wall composed of peptidoglycan except Mycoplasma ugar backbone (glycan) + peptide side chains nspeptidase + peptidoglyca (Peptido) cross-linked by trar Peptidoglycan structure ry Bs Osan nsonat cmt tt me the entry of essential substances such as sugars, amino ‘Such as penicilins. Lysozymes are enzymes that kl bacteria by cleaving which | parca the cll wai | PA Pentagyein bridges B. Multiple layers of peptidoglycan “net” CB 14 glycosidic bond between NAG and NAM | D.Tetrapeptide side chains GRAM-POSITIVE VS GRAM-NEGATIVE, C5 Aptis yams and metals as wel as many animirobial drugs Qe cee cee | folowing EXCEPT which one? ‘COMPONENT GRAM[s) CELLS GRAM (-) CELLS yaapue asl Thicker Thinner, B. Immunity Peptidoglycan multilayer thinner layer C. Circadian rhythm | Teicholc acids ‘Yes ‘No | Di Steep [Lipepotysaccharide No Yer | Which prion-elated disease Is caused by mutations Inthe PrP | Perl ce__No Yes ? Op Kuru GRAM-NEGATIVE CELL WALLS Pe aiopease co | outer membrane of gram-negative bacteria cootans endotoxin © Familat cio | ipopolysaccharide) | pBse © consiits of lipid A and O antigen -> induction of I-1 & TNF TOPNOTCH MENICA! ROARN PREP MICRORIOI OGY HANDOUT BY PACIFICO ERIC E. CALDERON. M.D, ‘+ All gram-positive bactaria have NO endotoxin EXCEPT Page 3 of 71 * 1 It ey TOPNOTCH MEDICAL BCARD PREP MICROBIOLOGY HANDOUT BY PACIFICO ERIC E. CALDERON, M.D. For inquiries, visit www.topnotchbeardprep.com.ph or https://www,facebook,.com/tepnetchmedicalboardprep/ GRAM STAINING COLOR CHANGES DURING GRAM STAIN, el ram Hegove BACTERIA NOT SEEN _ ALTERNATIVE MAME REASON coeenneny Toa meh pid in col yeobacterioe " wallso dyecannot ACID-FASTSTAIN penercate ‘BARKFTELD Spirochetes + Toothintasee eeeecoee ‘Mycoplasma + Noell wall NONE sp. + very small (SEROLOGIES) Legionella + Poor uptake ofl su yep stain BB. counterstain + Intracellular GIEMSA STAIN > Cramydiog very small’ INCLUSION BODIES ioe Hotracelilar GIEMSA/ TISSUE beanesie very small STAINS. REQ ‘BACTERIA NOT SEEN IN GRAM STAIN ‘These Rascals May Microscopically Lack Color. , Treponema | Rickettsia ‘Mycobacteria | Mycoplasma Legionella Chlamydi | Which component/s of the Gram-negative cell wall is/are responsible for Its nonspecific endotoxin activity? i ‘A. Lipopolysaccharide |B. Telchate acid © Peptidoglyean D. Enzymes in the periplasmic space Which of the following is the OLDEST acid-fast staining i | method, which requires heating the specimen during the | Procedure? \. Fluorochrome B. Ziehl-Neelsen, hodamine ‘of the Gramienegative coll wall reduces its ‘susceptibility to lysozymes? i A. Telchole acids [cetWal TAs described previously [cyteplaemic [Upoproteln bilayer uke of xdative and Jmembrone _|without sterols‘ ___|transpert enzymes _ [RNA and protein in SOS [Protein synthesis land 308 subunits rmaceatd [DNA "———Jaenanemitorat — invagiaaion pase —ourticpate in ell dion [Mesesome | membran land secretion [Space between plasma |Contalas many hydrolytic membrane and outer enzymes, including B- membran cramases NON-ESSENTIAL COMPONENTS OF BACTERIA [capsule [Polysaccharide _ [protects against phagocyiorls [pias or Io, Limace _(GheepreteIn [Attachment conjugation [iycocapa [Polysaccharide | Wediaes adherence towurisces Fegellun [Protein IMouiey Keratiila coat spore stance to est, and chemicals a lplclinicacig _[A**tance to Neat and chemical ame [Genes for anibioti resistance and roxne [aveouen. pas, renule site of nutrients in cytoplasm [polyphosphates eee + Allbacterlal capsules are composed of polysaccharide EXCEPT Bacillus anthracis (———— ) + Spore: formed by gram-positive rods, especially Bacillus and Clostridium spp. ‘SPORULATION IN BACTERIA (Otamenemietns rn TOPNOTCH /AEDICAL BOARD PREP MICROBIOLOGY HANDOUT BY PACIFICO ERIC E. CALDERON, M.D. For Inquiries, visit www.topnotchboardprep.com.ph or https://www.facebook.com/topnotchmedicalboardprep/ Page 4 of 71 PLASMIDS atrachromosomal, dout replicating independently of the bacterial chromosome. * Can sometimes be integrated Into the bacterial chromosome > called eplsomes Slgnincance + Antibiotic resistance * Resistance to heavy metals « Resistance to UV light + Plt (Ombriae) ‘+ Bxotoxins and several enterotoxins, «+ Bacterlocins - oxic proteins produced by certain bacteria that are lethal for other bacteria, «+ Nicrogen-fxing enzymes in Rhizabluim inthe root nodules of legumes ‘+ Tumors caused by Agrobactertum in plants + Saveral antiblotics produced by Streptomyces, radative enzymes produced by Pseudomonas capable able of cleaning up environmental hazards such as cll spills ‘and toxle chemical waste sites > I, BACTERIAL GROWTH + Bacterial groweh sa coordinated process of increase Ip Individual cell mass and size and duplication of the ‘chromosome, followed by cal division, * Bacterial reproduction occurs vi binary fission Bacterial growth cycle hte ite PHASE 1: LAG PHASE * Celis are depleted of metabolites asthe result of unfavorable condition ‘Adaptation to new environment «Nutrients are incorporated * Vigorous metabolic activity occas but cells do not divide + Zero growth rate PHASE 2: LOG OR EXPONENTIAL PHASE * Rapid cell division occurs SeLactam antbiotis act during this phase * Constant groyth rate + Continues yl either one or more nutrients in the medium ‘become exhausted, or toxic metabolites accumulate and inhibit growth, PHASE 3: STATIONARY PHASE ++ Exhaustion of nutrients or the accumulation of toxic products cause growth to cease completely (zero growth rate) «# Spores are formed PHASE 4: DECLINE OR DEATH PHASE + Most ofthe cells die because nutrients have been exhausted + Negative growth rate ‘TOPMOTCH MEDICAL BOARD PREP MICROBIOLOGY HANDOUT BY PACIFICO ERIC E, CALDERON, M.D. -atranded, circular DNA capable of :sTOPNOTCH MEDICAL BOARD PREP MICROBIOLOGY HANDOUT BY PACIFICO ERIC E. CALDERON, M.D. , or Inqutres, visit worw.topnotchboardprep.com.ph or htips://www facebook.com/topnotchmedlealboardprep/ SFFECT OF BACTERICIDAL AND BACTERIOSTATIC ANTIMICROBIALS a Catt seems dg come a Pre sya tees 1 L 7 vetoes uae as IV. BACTERIAL OXYGEN METABOLISM ‘AEROBICAND ANAEROBIC GROWTH ‘+ oxygen metabolic generates toxic products Zhch as superoxide ‘and hydrogen peroxide * i ES vantemc eavironments he ENZYMATIC DETOXIFICATION, _ ‘uperoxise lamutase Oy +2Ht —> H,02 + Oz colalese 2,02 >» 2Hz0 + Oz H,0, + 2 0+ SHH, 2 HL ‘AEROBIC METABOLISM * Obligate Aerobes ‘completely dependent on oxygen for ATP-generation + Microaerophiles . ‘use fermentation but can tolerate low amounts of oxygen because they have. ‘ANAEROBIC METABOLISM ‘+ Facultative Anaerobes ‘utilize oxygen iit is present, but can use fermentation in its absence + Aerotolerant Anaerobes exclusively anaerobic but insensitive tothe presence of oxygen + Obligate Anaerobes © cannot grow inthe presence of oxygen because they lack 3 Important enzymes: and Page 5 of 71 For inquiries, visit www.topnotchboardprep.com.ph or https://www.facebook.com/topnetchmedicalboardprep/ rina yw facebs a ae For Inquiries, visit www.topnotchboardprep.com.ph or https://www.facebook.com/topnatchmedicalboardpreP! 1 BACTERIAL OXYGEN METABOLISM. a ierooerophies? } ore wont —Trecroromen |_| Whshenyme emigrate icone 1 bigaeaeabe "Growin | No Growin [naguves 8. Catalase ' ncronerophie | crown | Norowin [Rtauedbut slow | cE i Beart pa OI a TOE caus | Not required for crtarismtc uve Inthe presence ofall amounts of | wave | rowan | orowth {growth bus utined xen, Which ofthe following ensymer catalyes the reaction bi | | hon eva le Sore the > Hoes Or? aroaen ; Not required nd or ‘Superoxide clmutase recess crown | crown [Seis Bese 7 =| Peroxidase ones —Ipeceown| crown hoe 1D Aboftese Co $e ar ain nee, —| BACTERIAL GENETICS ‘revamone, bora eo, Oblate Aerobes | srr Aheteciter en DWA TRANSFER BETWEEN BACTERIAL CELLS a + Taster Type of Cale Sapo procera rs ] a Microzerophies | reponemes nonusers teheer brocade proces || nv | perros tec entrec, | DNA Taniered Tem Only rar acie |Facutatve Anaerobes | ewvsecrnon ttc hraneimenia Coniston |acterumto another [AN . = | DNA transferred by a Aerotolerant Propiontbocterlm, tactobocus Transduction [vrstrom one calito | Prokarotc [anaerobes | Ms NA rotary or siete sects | sage oi saan [ORREONR ND PE ‘Nosy and Nagging Bests Must Breathe Lots of oxygen. ‘Nocardia —_._} | Nelsseria ace re | Bacteroides | = _ [TRANSDUCTION = snageona O ooce ares rte Om LO Becterial | a | 4 | AND) | one ® 7 Lies releasing: © ars * SCY | © Wow phage OnA we © Phage OnA ingrates win he | oe a a CS ‘TOPNOTCH MEDICAL BOARD PREP MICROBIOLOGY HANDOUT BY PACIFICO ERIC E. CALDERON, M.D, Page 6 of 71 Fer inquiries, visit www.topnotchboardprep.com.ph or htips://www.facebook.com/topnetchmedicalboardprep/ TOPNOTCH MEDICAL BOARD PREP MICROBIOLOGY HANDOUT BY PACIFICO ERIC E. CALDERON, M.D, For inquiries, visit www.topnotchboardprep.com.ph or https://www.! jebook,com/topnotchmedicalboardprep/ “GRIFFITH EXPERIMENT - TRANSFORMATION ((ournal of ynlene 27 (Gy 113-189, 1928) CTI — CD Be bd 1 gl imo mouse Injected inte mouse: (an rw <3 Ge 3) ae Ne tp Sow) 1 bacterlophage introduce DNA to the recipient bacterium? {A Transduction B, Transformation © Conjugation . Transposons {In which form of form af genetic exchange does a | | | | [ ‘VI. NORMAL FLORA ~ microorganisms that are the permanent residents ofthe body ‘+ normal flora are low-virulence organisms in their usual anatomic site + _—_oceurs when normal flora occupy ‘Feceptor sites preventing pathogens from binding Sin Staphylococcus Nose ‘Staphylococcus — —— Mouth Dental plague: = ‘Colon Bacteroides Vagina eon Screptococeus egalaction INFECTION BIOLOGY PATHOGENESIS ‘= pathogens are microbes that can cause disease; opportunistic pathogens cause disease only in immunocompromised people + virulence is a measure of a microbe's ability to cause disease + determined by virulence factors, such as capsules, exotoxins, or endotoxins + 1Dsos the number of organisms required to cause disease in 50% of the population BACTERIAL INFECTION + Infection has two meanings: (2) the presence of microbes in the body and (2) the symptoms of disease + presence of micrabes in the body does not always result in symptoms of disease MECHANISMS OF BACTERIAL DISEASE + production of toxins (both exotoxins and endoto» ‘Induction of Inflammation TOPNOTCH MEDICAL BOARD PREP MICROBIOLOGY HANDOUT BY PACIFICO ERIC E, CALDERON, M.D. For Inquiries, visit www.topnotchboardprep.com.ph or https://www.facebook.com/topnotchmedicalbaaruprep/ MODES OF TRANSMISSION + Human to Human ‘o direct (sexual, transvaginal), fe:el-oral, inhalation, transplacentai, blood-borne ‘+ Nonhuman to Human ‘soll, water, direct animal source, vector-borne, animal excreta, BACTERIAL ADHERENCE * plll mediate attachment of bacteria + alycocalyx mediates strong adherence to surface of human cells ‘surface proteins called curl! (Salmonella and B. colf) mediate binding to endothelium and to extracellular proteins suchas yronectin, Gu Salma Hayek and her curif hair (Salmonella) ENZYMES IN BACTERIAL INVASION + collagenase and hyaluronidase co spread through subcutaneous tissue * coagulase ‘accelerates formation of fibrin clot coating the organisms ‘with a layer of fibrin ‘+ immunoglobulin A (IgA) protease allows adherence to mucous membranes ‘o destrays both neutrophilic leukocytes and macrophages ‘Which ofthe following may be considered primarily responsible for the developmentof cellultis from a small furuncle? ‘A Lipases B. Hyaluronidases ©. Panton-Valentine leukoctdins jemolysins Bacterta with Protease, | stave my Gong | "Streptococcus pneumoniae | Haemophilus influenzae Nelsserla Meninghidis Neisseria Gonorthoeae Pag Tot 71 es VIRULENCE FAcTons + Polysaccharide Capsule S tnucapalarandvotiey im ® anucapalar antibodies allow more effective phagocytes ‘occur (OPSONIZATION) - ea + Coll Wall Proteina ‘OM protein ofS nyogenes of antiphagocytc S aureus prevents complement ‘atvatlon ~~ ésia5in — poilvesnd war Ineguivebacaria |lexcePr ephyiccocea! 'stbiewt 100rctors hour | enterotoxin) 2 ee i Tewnus vote ~~ [warigsesecanin EXOTOXINS '= polypeptides secreted by certain bacteria thal alter speci functions resulting inthe symptoms of disease * have an A-B subunit structure: ‘A subunits the active (toxic) subunit ‘0B subunit isthe Binding subunit MECHANISMS OF ACTION OF EXOTOXINS:~ [Diphtheria toxin, cholera toxin, Escher collheatlablle toxin, and pertussis toxin Hoxie shock syndrome toxin, staphylococcal super Permigen enterotoxin and erthrogei toxin etenus tein botulinum ts etal cor _Proweate lot anthrax toxin, and salded skin toxin Teaahinase —|Glstrdlam peringesspha tocn EOS ‘When an exotoxin binds toa receptor, it activate T-cells by " binding SIMULTANEOUSLY to a T-cell receptor and MHC Il | molecule on an antigen-presenting cell WITHOUT requring an antigen. Itactivates large number of T cells to cause a cytokine storm, What isthe nature ofthis exotoxin? A. Superantigen B. Protein synthesis inhibitor C. ADP-ribosylator D. Protease ‘Which ofthe following kills cells iby eevng 6S osama | subunits? ‘A. Bxotoxin A B. Shiga toxin C Amikacis i D. Clindamycin ‘TOPNOTCH MEDICAL BOARD PREP MICROBIOLOGY HANDOUT BY PACIFICO ERIC E. CALDERON, M.D. For inquiries, visit www.topnotchboardprep.comn.ph or https://www.fac ‘TOPNOTCH MEDICAL BOARD PREP MICROBIOLOGY HANDOUT BY PACIFICO ERIC E. CALDERON, M.D. For inquiries, visit www.topnotchboardprep.com.ph or https://www.facebook.com/topnotchmedicalboardprep/ Which ofthe following bacterial toxins is neurotoxic? i 4. Listeiolysin © B. Anthrax toxin G Shiga-ike toxin . Tetanospasmin } ENDOTOXINS, * lpopolysaccharides (LPS) located in the outer tembrane of gram-negative bacteria + Liptd A is the tonle component of LPS © induces the overproduction of cytokines, such as TWF && IL-1 ‘activates the complement cascade ‘activates the coagulation cascade, resulting In disseminated Intravascular coagulation MEGAMI endotoxin ‘eNdotoxin i an integral part of gram-Negutive cell walls, i _ noon Cad 22 [GENERALITIRS ON BACTERIAL STRUCTURE ‘all bacteria have cell wails composed | Mycoplasme | of peptidogivcan EXCEPT pneumoniae | Usterio | monocytogenes ‘Ail gram-postive bacteria have NO fendotoxin EXCEPT ‘All bacterial capsules at composed | Bacillus ‘of polvsaccharide EXCEPT | anthracis All exotoxins are heat-abile excep | St@Phylococcal enterotoxin Wrenner ‘MICROBIOLOGIC EXAMINATION * Direct Examination ond Techniques + Culeure + Microbial Identification: Colony and cellular morphology: ‘growth characteristics under various conditions, utilization of carbohydrates and other substrates; enzymatic activity, immunoassiys, and genetic probes + Serodiagnosis: A high or rising ter of specific IgG antibodies or ‘the presence of specific IgM antibodies may suggest or confirm ‘a diagnosis + Antimicrobiol Susceptibility: Microorganisms, particularly bacteria, are tested in vitro to determine whether they are susceptible to antimicrobial agents Page B of 71 book.com/topnotchmedicalboardprepy oinieiienaemmanandiaa -gigq) TOPNOTCH MEDICAL BOARD PREP MICROBIOLOGY HAND JOUT BY PACIFICO ERIC E. CALDERON, f4.D, For inquiries, visie www.topnotchboardprep.com.ph or htips://www. face book.com/topnatchmedicalboardprep! LABORATORY INVESTIGATION IN DIAGNOSIS OF INFECTIOUS AGENTS. Paton bevrmin lage lero! I Selec nd cotae mou appropriate spacimant ‘grins wicca agora sonpoca 1 __ ‘REET DETECTION BY. ‘cuLTURE Space sain (ug. Gram, tdortzaton by osname Besant Cleo: wt) ‘aechone, dh orale 98 ‘arvnorialisoaeaton ae | | | | gc ; = eens gmorizee ron noroete te [ruein Manat cae ta diagnos fv nection wh logncbaton anne |ciGcoUaTS period prio wo the appearance of nical manifestations eens fain evocoure + “pin erred pais vise. a HIV [Herons nuerne _|sacronsxondv _} sey tetingfor abodes thee vires ithe fit ep 1 owed nar yma aclsampiieson Te Tecra aid * [tected] uuor uoREScENT (uF ANTIBODY STAMNING es aac a |pbeeitecen [eben eons | re pce than ther sang technique butalso ore Se cmrmtaglecaa |" nisaceme spake orders sere perl « Conmonl es hureseln labeled anoles careshyet cae Tepes pn «Garant aaingen specimen rom poets smore Skirrows, _ ‘Compylobacter, Helicobacter | dificult and less specific : ceeen ae pvr pertusorLginlpreumophein Sees Rero ay BR] [aera bose Common sie: Sordwale pre |Eaton Mycoplasme pneumonioe (A direct immuretuorescence B ingirect immunoliverescerc.e eeu rendonaes sre iow ove Db] | Seon, Sie Ether cileuprihmoe : [Harris (EMJH) / Fletcher's “ Interrogens MOLECULAR TESTS Sods anpatan ts RTOS Madeeponce nal nian “imma A nse «» highly specific, quite sensitive and much faster than culture test, the fluorescent dye is atta¢ sNeygetcnimmemndm trem, LLL) LLL | secodemerane sizhasChomyale and nyeobactertum species wiry UTP meesenee Wu AAA aint iniec Arsen antioty i tanceeeetayiaces a sentbogmeeapinan A. Diet torescent 6, nauecttucacan 8G nth est ‘oop et TOPNOTCH MEDICAL BOARD PREP MICROBIOLOGY HANDOUT BY PACIFICO ERIC E, CALDERON, M.D. Page 9 of 71 Fre inaniriae viet wrw tannatchhaardaran crm nh nr hitne: duu facahank eam/tannatrhmedicalboardoreo! For inquiries, visit www.topnotchboardprep.com.ph or https://www.facebook.com/topnotchmedicalboardprep/ “FLO TOPNOTCH MEDICAL BOARD'PREP MICROBIOLOGY HANDOUT BY PACIFICO ERIC E. CALDERON, W.D. RABIES IFAT ables antibody used for tastis primarlly directed agalnat the nucleoprotein (antigen) ofthe virus. «© Rables virus replicates inthe cytoplasm of cells, and infected. cells may contaln large round or oval inclusions contalning collections of nucleoprotein (N) or smaller collections of antigen thatappear as dust-like fluorescent particles i stained by the dFA procedure. ‘SEROLOGIC TESTS FOR SvPHILIS + Nontreponemal screening tests, easy to perform, low cost fe Venareal Disease Research Laboratory VDRL) «require microscopic examination to detect locculation ‘Rapid plasma reagin (RPR) co Antigen: cardlolipin cholesterol and purified lecithin ‘© Others: Unheaced serum reagin (USR) and tluidine red unheated serum test (TRUST) + Treponemal antibody tests— for confirmation © measure antibois against 7 palidum antigens © Tpalidum enzyme immunoassay (TPEIA) fuisaor EA 1s Coathe walls of plane ierodilon way with aed antigen ~~ 2. Add specimen an cube and thn wah he wel Shade ed andbodios Wash, arg ands are presen fssumed thatthe enayme-beled anil be Sound to te once te subaru adda te eye wl exaae he, formation aor eh 5. Aaaie color angel spectrophotometer. ‘Advantages of indirect ELISA + More sensitive: Ech primary antibody contains several epitopes that can be bound by:the labeled secondary antibody, allowing for signal amplification + Maximam immunoreactivity ofthe primary antibody Is, retained because itis not labeled. + Different visualization markers can be used with the same Primary antibody RIA * Similar with ELISA but uses radio-labeled antibodies NOT enzyme . + Instead of color change measured by spectrophotometry, uses, ‘gamma counter to measure radioactive signal = © sige > ‘TOPNOTCH MEDICAL BOARD PREP MICROBIOLOGY HANDOUT BY PACIFICO ERIC E, CALDERON, M.D. -/Iveww facebook. com/topnotchmedicalboardprep/ For inquiries, visit www.topnotchboardprep.com.ph or htt LATEX AGGLUTINATION + Primarily directed atthe detection of carbohydrate antigens of ‘encapsulated mleroorganisms + Steps: 1 Antigen-specific antibody (elther polyclonal or monoclonal) In fixed to latex beads, 2, Add clinical specimen. 43. the target antibodies are present, they bind tothe ‘antigens on the microorganisin forming a lattice structure, and agglutination ofthe beads occurs a -Y— npcomes — Aemmeey Seon a Aoqmane we: pace YY » + Uses: dlagnosis of grotp A streptococcal pharyngitis, detection, of eryptococcalantiggn (CALAS), bacterial meningitis WESTERN BLT IMMUNOASSAYS formed datec antibodies against specific antigens + Based. spon the elocerophoretic separation of major proteins of ‘hegrdigion in que na two-dimensional aarose 1c LoPathogens are mechanically or chemically disrupted and resultant solubilized antigen of the pathogen i placed Ina polyacrylamide gl. 2. An elecri current is applied and major proteins are separated out onthe basis of size (smaller protzins travel faster) Proteins are transferred (blotted) onto the surface of 2 smrccmose | membrane ad cdi a ik Lull ‘The membrane cout into strips, ‘Svips are incubated with palient serum and entihuman IgG conjugated with an ‘enzyme (and chromagen) Page 10 of 71 + Jlrsergus: ToPNoteH MEDICAL BOARD PREP MICROBIOLOGY HANDOUT BY PACIFICO ERIC E, CALDERON, WD, or inquiries, visit www.topnotchboardprep.com.ph or httpst//www.fecebook.com/topnotchmedicalboardprep/ Mas lnone wma Techn seem saree pp emcee aeons rere aaate eee eee ene | & feevoa ©. Eosin-methylens blue agar ue | D. Middlebrook 7H20 agar ‘Which ofthe following molecular techniques represents eae or the polymerase ‘chain reaction, involving the use | ofan enyme to conver vir RNA sr mewenget, prior to amplification? bat I Srandhedseain DNA aay C. Reve j |_ skeen | PARADE OF ANTIMICROBIALS Ea SR In the diagnosis of IV, which technique employs tips that | are incuba with patient serumandantsumen gC 1000-BRAIN BARRIER PENETRABILITY~- MEMORIZE ‘A. Western blot ‘ Good only with 5 FISH | excellent with orwithout | Inflammation C: DNA Microarray » {nftammation Peniclins D. Southern blot ‘Sulfonamides Cephalosporins: cefuroxime ; — Chloramphenicol (2-4 gen); 3-4 gen parenteral Gideon, 29/M, consults for three weeks cough [eady Trimedhroprin (except elaperazone) ro sputum. The physician suspects that ke has' sand | Metronidazole aa fequest for sputum examination to visualyg Bigg ast Ronen —— bacteria, Which of the following theo thes: | ae ees fast staining methods, the one that reqi icing the | Fiuconazole aateecmen specimen during the procedure? Flucytosine Ciprofexacin “A BiehlNeteen | Vancomycin |B Kdnyoun uramine-rhodam Winimal or not goodeven | No passage even with C Aaramtanriene |] Sid ntammation inflammation - a Aminoglycosides: Polymixins: “an 16 year-old malgpeseits with urethral discharge You | Taneeio oid gia suspect he has agqaytoccl| infection. What is'the gold Lincosamides cephalosporins | standard for didgno8ls? Macrolides Amphotericin B | ad ‘and Gram stain B. Cul colate agar with factor X and V MECHANISMS OF ANTIMICROBIAL RESISTANCE © Calture on Thayer-Marain medium ‘nanre §@ D.Unnaysis ‘Which isthe mostrapidand specie west for Mycobacterium tuberculosis?” | A. Léwenstein-Jensen medium B. DNA amplification . APB salning i D, Ginel diagnosis | 7 hich ofthe folowing molecular wechniques represents a variation of the polymerase chain reaction, involving the use| | ofan enzyme to convert viral RNA or messenger RNAtO DNA | prior to amplification? i | A Reverse transcriptase { B. PCR Real-time PCR © Branched-chain 1 D, DNA assay Southern blot | L_BLOMA assay Southera blot ¢ ‘TOPNOTCH MEDICAL BOARD PREP MICROBIOLOGY HANDOUT BY PACIFICO ERIC E, CALDERON, M.D, Page 11 of 71 BL cy Tornoren hepicas soano ener wicrosio.o6y HANDOUT PACIFICO ERIC E. CALDERON, M.D. For Inquiries, visie vrw.topnotchboardprep.com.ph or https://www.facebook.com/topnotchmedicalboardprep/ RESISTANCE IN SOME ANTIBIOTICS + Belactams Hydrolysis, mutant PAP + Tetracycline ‘Active efflux from the cell + Aminoglyeosides Inactivation by enzymar + Sulfonamides Overproduction of target ‘+ Flugroquinolones ‘Mutant DNA gyras * Chloramphenicol Reduced uptzke into cell, ‘Vancomycin Reprograming of D-aia-D-aln + Quinupristin/ dalfopriscin; Ribosomal methylation {+ Macrolides Erythromycin: RNA methylation, drug effux ning shortness of iga dlastolle murmur yieal history is i Bignileant fora lef hip replace reago,andshe ‘vaguely recalls an extended asa child I ‘deneribed as several severe episodes of sore throat followed + by rash, (ever, and joint palns. Administration of which of the | following treatments at that time would have been most | effective in reducing her risk of developing cardiac disease? ‘Penicillin | B, Vancomycin © Cipreffoxacin i D. Tetracycline | When combined with the beta: sctamase inhibitor 1 tazobactam, this penicillin posserses the broadest | "A28 year-old Caucasian male with cystic fibrosis developed Pseudomonas aeruginosa pneumonia. Sensitivity studies showed resistance to levofloxacin. Which of the following drug, resietance mechanisms is moat lkely responsible for this? ‘A, Production of BSBL B, alteration of yyrases | C Production of acetyleranferases D. Decreased ribosomal binding Sulfonamides may Increase the risk of neonstal kernicterus by | Which mechanism? i ‘A. Diminishing the production of plasma albumin |B Competing for blirubln-binding sites on plasma albumin |! Myelosuppression | _D. Increasing splenle sequestration of feal-yoe RBCs MUST-KNOW TC «+ Etlologic agants based on epidemiology, symptoms, or Information about organism. + Predisposing conditions and risk factors, mechanism of pathogenicity, major tests + Morphology «= Physiology «+ Soructures «+ Determinants of pathogenicity + Epidemiology/transmission Giteacterl specrum smongtsidnd-Which anukacterialis | Leperatory serbed? iS btceases a: Ampietin [ Strermen |S Amonetin |S Prwendan |G Piperaciilin I |B. Tearain. penatomore Bete . = cel ston epi, | ces a aos Tg nse Sweneeee ce oe ‘cower oon © Lineoge ae Denne TOPNOTCH MEDICAL BOARD PREP MICROBIOLOGY HANDOUT BY PACIFICO ERIC E, CALDERON, M.0.. Page 12of Ty For inquiries, visit waw.topnotchboardprep.com.ph or https: / www facebook.com/topnetchmedicalboardprep! “Faw TOPNOTEH MEDICAL BOARD PREP MICROBIOLOGY HANDOUT BY PACIFICO ERIC E, CALDERON, H.D. . ' For inquiries, visit www.topnotchboardprep.com.ph or https://www.facebook.com/topnotchmadicalboardprep/ Gram-Negative Bacteria Dipiacocet, oceabaci seca Commashaped o een Aaroble [| | oF ‘ae secs se aeaan mt a = A ee ewatows pacman Fi yo eee Lactose fermenting 1H, S-producing — tao mem Lineage | A newborn develop regpiratory datess and lethargy. Blood | lure grew Gram postive, catalase negative, beta hemolytic | bacitracin resitan colonies. Which the most likely etlogy? ‘a Sepeococeus pneumoniae t 8. Smeptococeuragolaclee j €. Seeprococeus pyogenes D. Streptococus mutans (Gansu) [Grim © ousiaive) Ji \ ‘TOPNOTCH MEDICAL BOARD PREP MICROBIOLOGY HANDOUT BY PACIFICO ERIC fr Has vitae trnnatchhaardaren com nh ar bos: //www fa (GRAM STAIN (OTHER FEATURES >, , am postive eoecin clusters |catalae postive, [conguse postive stale postive, coogulase negative, |novebiocin sensitive catalase postive, coagulate negative, Inovoblocin resistant ‘Staphylococcus saprophyticus E, CALDERON, M.D. jbook,com/toonotchmedicalboardprep/ Page 13 0f 71 > _—_—$_. ‘A. STAPHYLOCOCCUS AUREUS CHARACTERISTICS + gram-positive coce! in grape-ke clusters + Bhemolytic yeliow or golden colonies on blood agar + catalase-positive + coagulase-positive * saletoleranton_ “Gold* color ls due the pigment HABITAT + human nose (anterior nares) and skin ‘TRANSMISSION + direet contact (hands) *fomites + contaminated food VIRULENCE PACTORS: IMMUNOMODULATORS. Protein * prevents complement activation ‘Congulase + bullds an Insoluble Abrin capsule Hemolysing ear * tort to heriatopoletic cells PV Leukocidin + specific for white blood calla Catalase + detoxifies hydrogen peroxide nelllinase _« inactivates penicllin derivatives [TiSSie PENETRANCE Hyaluronidase + hydrolyzes hyaluronic add Fibrinolya + dissolves fbrin clots FOPNOTCH MEDICAL BOARD PREP MICROBIOLOGY HANDOUT BY PACIFICO ERIC E. CALDERON, M.D. For Inquiries, visit wwrw.topnotchboardprep.com.ph or hitps://www.facebook.com/topnetchmedicalboardprep/ ‘SPECTRUM OF DISEASE: PYOGENIC © bullous Impetigo oliculitis, furuncles carbuncles, cellu, mastitis, surglal site Infections ‘© most common cause of acute endocarditis native valve valve) In IV drug © from hematogenous spread or local introduction at wound site . abscess focus of osteomyelitis arising in the ‘metaphyseal area ofa long bone SPECTRUM OF DISEASE: ToxrGRNIC + Gasteaenterits ‘acute onsee(¢hr) of vomiting and darren due to Ingestion of preformed heat-stableenterotoxin o soure: salad made with maygnnaize (potato o tun * Scal ed in Syndrome (Ritter disease) exfollatin cleaves, f ‘separation of epidernirar- © distinguish from TEN (Lyell disease) where separation occurs at dermovepldermal junction In desmosomes ‘odue to TSST-Y © fever, hypotension, sloughing of the fliform papillae > strawberry tongue, desquamating rash ang multi-organ * causes marked necrosis of the skin and Alphatoxin ere eI Ciesietehy ‘TOPNOTCH MEDICAL BOARD PREP MICROBIOLOGY HANDOUT BY PACIFICO ERIC E, CALDERON, M.D. staphylokinase) fnvolvement (3) Lipase + spread in fav-containing areas ofthe usually no site of pyogente inflammation; blood CS negative bod J ‘usual scenario using menstruating ‘women or In palerts with nasal packing for epistaxis Frexne1__ TREATMENT cattle + causes epidermal separation ‘methiclllin-sensitive — + penicillinase-resistant penicillins inerotoxing SA(MSSA + (nafeilin, exacilin, & diloxacilin + superantigens causing food poison i. rihenestable)_* peronigene ese Cot potoni_ | ‘methicilin-resistant —» contain altered PBP To hock ng superanigen leading to tove shock sacs + DOC vancomycin syndrome vancomyela . (rsst-1) TeataneSavnsa) —_* 20Cislinezld | A6d.yearcd female recovering Fom oil nuder | arhoptsy completed 6 dye ago presents complain of Jon pine herrepatred shoul, Temperssre 39 degrees | Cute Payee examination demons erynems snd | Suan tenderneas around the inci ste Wound Cultures revel Grampostve coed that are ean fo | tafe Which ofthe following organisms the mos key Cause ofthis pate conon? 1 Srepococus ridans 2 Suaphlococeuseplderids C. Staphylococcus aureus | 0 Srepeccsapyageet Tatlin esistance in Saphyloaeersorar caied Th wich ene? 1 er 2 mech care Doge | Rieyearold Galan wale presen wih pani trytormabous ned honey colored cstedlsions around ia auth Clare ofthe leon eves gra-potive coc in clusters, Further analysis reveals bacteria that are beta- | | emolytcotguase postive, and appear golden cn the lood. | | agar plate. Which of the following helps the bacterium in this | infection bind to immunoglobulin and prevent phagocytosis | hen imadng shot? | "A rink 2 Suphyloliase ) C. Exfoliatina D. Protein Page 14 of 71 For inquiries, visit www.topnotchboardprep.com.ph or https://www.facebook.com/topnotchmed ROE 1B STAPHYLOCOCCUS EPIDERMIDIS. ‘CHARACTERISTICS + gram-positive voce! In clusters + catalase-positive + coagulase-negative + novoblocin-sensitive ‘whitish, non-hemolytic colonies on blood agar UII ovobocin sonst ‘NO StRES 1 sores \ SaprophgeensReinant i idermidis Sensiave J — HABITAT + normal skin fora ‘TRANSMISSION ‘+ auroinfection + direct contact (hands) PATHOGENESIS + low-viralence organism + glycocalyx adheres well to, ‘o prosthetic heart valves, ‘o prosthetic joints ‘oventriculoperitoneal shunts ‘o indwelling catheters SPECTRUM OF DISEASE ‘+ most common cause of ‘oprosthetic valve endocarditis ‘Septic arthritis in prosthetic Joints oventriculoperitoneal shunt infections ‘TREATMENT ‘+ removal of prosthetic device + over 50% are methicilin-resistant and thus require vancomyein . ‘CSTAPHYLOCOCCUS SAPROPHYTICUS ‘CHARACTERISTICS « gram-positive cocci in clusters ‘= catalase-positive + coagulase-negative . + novobiocin-resistant + whitish, non-hemolytic colonies on blood agar « Nirite negative (unlike E. colt) EPIDEMIOLOGY ‘ i most common cause of UTIsin sexually active women CLINICAL FINDINGS « dysuria, pyuria, and bacteriuria ‘TREATMENT ‘© TMP-SMX, quinolones STREPTOCOCCI - OVERVIEW a [aRAMSTAIN | OTHER FEATURES ORGANISM, eaalae renee re |aiphahemalvic, | meumonise |pile-optochin-sensitive [estalase negative, |alpha hemoivtic, [bite-optochin-esistant catalase negative, ewes | streptococcus [escnmcingenstve_| PPO#EMEE | ‘catalase negative, | seraprococcur | beta hemolytic, ‘agalactice bacitracin resistant viridans streptococci [gram positive [cocelin chains For inquiries, visit www.topnotchboardprep.com,ph or https://www, | [catalase negative, | group 0 streptococci |pamma hemolytic TOPNOTCH MEDICAL BOARD PREP MICROBIOLOGY HANDOUT BY PACIFICO ERIC E. CALDERON, M.D. iDOUT BY PACIFICO ERIC E, CALDERON, M.D. book. com/topnotchmadicalboardprep/ ‘x Srmeprococcus ProGeNes CHARACTERISTICS + aram:postve oct in chalns + Grales-mnpaive 1 becoshemebyte 5 bactracn senate 5 Lancet group + pave ca The PYR tet measures hydrolysis ofp rolidont-- ‘apheylamide an sane of naphylmn, hh 2 the presone of peimetyiaminocenamslaehyde forme res compound. co Advantage takes les than 2 mite to determine whether the reaction ls positive (S. pyogenes or group D streptococci) or negative (all other streptococci). Bacl sensitiv ‘B- BRAS | Bacitracin group B strep Resistant, roup A strep Sensitive™ 7 HABITAT . « buman throat (oropharynx) .°, skin : ‘TRANSMISSION « respiratory droplets. patHocenssisy* *, VIRULENCE ENZYMES degrades hyaluronic acid + activates plasminogen > degrades DNA In exadates or necrotic todomnase) _ tissue ‘=[[CSA peptidase inactivates complement CSA (Toxins: Erythrogenic toxin + produces scarlet fever ‘Streptolysin O ~ highly antigenic (onygen-labile + causes AB formation ‘Streptolysin S (oxygerrstable) /ogenic exotoxin A+ superantigen similar to TST. [ye ‘protease that rapidly destroys tissue PATHOGENESIS: Evidences of Infection « antl-streptolysin O (ASO) titers to document antecedent pharyngitis 2 titers to document antecedent skin infection ‘oStreptolysin Os irreversibly inhibited by cholesterol in ‘kn lipids + anti-sereptokinase antibodies decrease efficacy of streptokinase in managing Ml SPECTRUM OF DISEASE: PYOGENIC ‘0 Impetigo contaglosa: ‘ perioral blistered lesions with honey-colored crust ‘accumulation of neutrophils beneath stratum corneum ‘complication: poststrep GN cobrysipelas ‘superficial Infection éxtending into dermal lymphatics oCellultets ‘o deeper infection Involving subcutaneous /dermal tissues o facilitated by. _—— (spreading factor) oNecrotizing fasciltls rapidly progressive infection of deep subcutaneous tissues o facilitated by exotoxin B Page 15 of 71 Ca ee i eta nens ficvas ‘o most common bacterial cause of sore throat ‘inflammation, exudate fever, leukocytosis, and tender CLAD ‘pyogenic complications: peritonsllar and retropharyngeal (Quincy) abscess, otitis, sinusitis, meningitis, ;PECTRUM OF DISEASE: TOXIGENIC ‘Scorlat Fever ‘© postpharyngitic deo eryehragent tox won nope sans ever strawberry tongue, canrfugl rath (sandpaper lie) PasdainesGesqusaton Sn Coe °. for susceptibility ‘clinically similar but milder than S. aureus TSS edusts pyogenic exotoxin A ‘© recognizable site of pyogenic inflammation © blood cultures are often positive ‘SPECTRUM OF DISEASE: IMMUNOLOGIC ‘opostpharyngitic taal ‘TOPNOTCH MEDICAL BOARD PREP MICROBIOLOGY HANDOUT BY PACIFICO ERIC E. CALDERON, M.D. For inquiries, visit www.topnotchboardprep.com.Ph or https://www.facabook.com/tupnetchmedicalboardprep/ 1B. STREPTOCOCCUS AGALACTIAE (GROUP B STREPTOCOCI)” ‘CHARACTERISTICS ‘+ gram-positive cocc in chains + Catalase-negative «+ Beta-hemolytie ‘ Bacltracin-resistant ‘+ Lancefteld group B «hydrolyze hippurate ‘= CAMP test-postive row using Lim broth HABITAT vagina ‘TRANSMISSION «+ transvaginal « transplacental SPECTRUM OF DISEASE “ * urinary tract infection in pregnant women "5, ‘neonatal pneumonia, sepsis and meningitis ‘0 cross-reacting antibodies to M proteins and antigens of ‘© most common cause <4 Joint, heart, and brain tissue “ee ‘0 predisposing factors Ale Jones Criteria: lntepartu fever (738), > ‘© migratory polyarthritis ‘© PROM (>16h) “ & pencare Sagal clans Su © erythema marginatum ‘ocomplemer ‘ © Sydenham chorea sendometitis x at © subcutaneous nodules. i ‘TREATMENT WEI _Jones criteria OCs pening .* i B-E-G-: peniilinG +'aminoglycoside for serious infections | Polyarthritis + all pregqarigwiomen should be screened for GBS colonixation / Eqrthema marginatum ‘45047 weskes 400 | Chorea (Sydenham) ‘oepersoprophylais with IV Penillin or Amplellin 4 hours i Caraitis (Pancardis) it BHF 9 delivery Subeutaneous nodules j xt T.CROUP D STREPTOCOCCI (ENTEROCOCCUS FABCALIS) Glomerulonephritis HARACTERISTICS ‘post impetigo (commonly the M12 type) OR postpharynitsé OM protein incites immune complex deposition on thes." tlomerular basement merabrane % ‘ant hypertension, prtaritl edema, hematuria ‘TREATMENT + DOC sPenicilinG i « patients with a history of rheumatic fever eg ig erm antibiotic prophylaxis to prevent recupancee! the disease uy Ree rareera gt daplin ape | thasore throat A throatswgb atare revealed gran ponte thar are sensivegspacrain The cds given | SeammentandsnthongSbany yee past | presents tothe emergertuepartment wit ysonea on | Eierconsed catstgd m whrgedcarsestbouere and | pulmonary edemaPhe patient's current symptomatology | ‘most likely ocrurred secondary to which ofthe followin ‘A Mole inimlery |B. type ypersenstvity | Typelilhyperseniivty |B. Leukocidin activity : Coagulase atv TOPNOTCH MEDICAL BOARD PREP MICROBIOLOGY HANDOUT BY PACIFICO ERIC E, CALDERON, M.D. Myatt wire taenatchnaardnran com nmr hiner fa Baar ntl «+ gram-positive cocci in chains + catalase-negative * gamma hemolytic colonies + Lancefield group D * bile and optochin-resistant + hydrolyzes esculin in blleesculin agar (BEA) + positive PYR test +E faecalis can grow in 6.5% NaCl while S. bovls cannot HABITAT ‘human colon * urethra and female genital trct can be colanized TRANSMISSION + may enter bloods ‘ream during gastrointestinal (GI) or genltourinary tract procedures ‘SPECTRUM OF DISEASES ‘© UTIs due to indwelling urinary catheters and urinary tract Instrumentation « billary tract infections «+ endocarditis in patents who underwent GIT surgery due Enterococcus faecalis . endocarditis in patients with ‘edominal malignancy due to Streptococcus bovis ‘TREATMENT + penicillin plus gentamicin ‘+ vancomycin for penieilin-resistance + linezolid for vancomyein-resistant strains Page 16 of 71 \ Pf ece qoPsoren weoicaL soano pase micnosioLocy HANDOUT By Pacinico Ene For nqutien, vat wor opnatcnbuaraprep Son oh ey PAID ERI Strapzococcus pyogenes bacterium | Bi Strepeokinase i ‘A M protein B. Modification of cell wall pentapeptide precursors | B, Streprolysin o ©. Sereptolysin S . Altered penicillin-binding proteins D._Reduced binding to DNA polymerase CCefalexin may be useful to treat which infection? ‘A. Enterococeal endocarditis, Bi Listerial meningitis, © MRSA pneumonia i D. Hidradenttis supparativa | B._Allofthe above 1D. STREPTOCOCCUS PNEUMONIAE CHARACTERISTICS * Gram-positive “lancet-shape * Catalase-negative + Alpha-hemolytic + Bile and optochin-sensitive + positive Quellung reaction I" cocel in patrs or chains cul ntachia Sensitivicy ‘OVERPASS ‘Oprachin Viridans Resistant ___Pneumoniae Sensitive Ei Some Killers Have Pretty Nice and Shiny Bodies. ‘Streptococcus pneumoniae Klebsiella pneumoniae Haemophilus influenzae Pseudomonas aeruginosa Neisseria meningitidis Salmonella typhi B group streptococci ‘Quallung Reaction | QUELLUNG reaction = capsular SWELLUNG (swelling) HABITAT AND TRANSMISSION + habitat is upper respiratory tract > wansmission via respiratory droplets PATHOGENESIS + copsule retards phagocytosis, + IgA protease for colonization cssubstance reacts with CRP ‘SPECTRUM OF DISEASES: PYOGENIC + Pneumonia - ‘most common cause of CAP « Blood-tinged, pink, or rusty sputum © Blood cultures often positive * Semtieshock ‘© splenectomy predisposes to sepsis ‘TREATMENT + Penicillin G «Levofloxacin or Vancomycin combined with Ceftriaxone for Penicillin resistance PREVENTION * polyvalent (23-type) polysaccharide vaccine * conjugated vaccine: pneumococcal polysaccharide coup'ed ‘with carrier protein (diphtheria toxold) TOPNOTCH MEDICAL BOARD PREP MICROBIOLOGY HANDOUT BY PACIFICO ERIC E, CALDERON, M.D, For Inquirles, visit www.topnotchboardprep.com,ph or https://www.facebook,com/topnotchmedicalboardprep/ CALDERON, M.0. .com/tepnotchmedicalboardprep/ posited in thelr lungs, deviate from i aria as oppoted toa typleal strain? le rolubility, B. Optochin senstuivity t ©. Quelling reaction D. smolysis on BAP. : E. VIRIDANS STREPTOCOCCI (S MUTANS, 5 SANGUIS) ‘CHARACTERISTICS ‘and optochin-resistant HABITAT and TRANSMISSION ‘habitat is oropharynx «enters bloodstream during dental procedures MEQ stte-optochin Resistance | searecnpeememge nan PATHOGENESIS lycocalyx enhances adhesion to damaged heart valves * protected from host defenses within vegetations: ‘SPECTRUM OF DISEASE ‘+ S: mueans, for dental caries ‘+S. sanguls, for subacute bacterial endocarditis (SBE) ‘9 moxt common cause of subacute and native valve ‘endocarditix = Scimermedius, for brain abscesses ‘TREATMENT + Penicillin G vith or without an aminoglycoside (Gentamicin) + Vancomycin for pentcillin-resistance * Linezolid for vancomycin-resistantsrains GRAM-POSITIVE RODS I. Bacillus anthracis ~~ . Bacius cereus WL Closeridium tetani IV. Glestridtum botulinum V. Clostridium perfringens VL Closeridium aifite Vil. Corynebacterium dipheheriae VIIL. Listeria monocytogenes [GRAM STAIN | _ OTHER FEATURES ‘ORGANISM aerobic, nonmatie, boecarshaped rebic, mati, reheated tried rice spore-forming me ae Positive anaerobic, [bulging can anaerobic, eethinaae | jensen; | clostcatum perting [aseerabie | chosscidivn aficte T serobie,nonmotie, | Cenmebacterium non-spore- eurved,chinese | diphtheriae forming chacseters, ' exam postive — a iserobic, curved, Usteria ltumblingmetiity | monocytogenes Page 17 of 71 eee Joos Jorworen mepien soano per enon Fatatiten etree LOGY HANDOUT BY PACIFICO ERIC E, CALDERON, M.D. 'p-com.ph er https://www.facebook.com/topnotchmedicalboardprep/ 1 BACHLUS ANTHRACIS TE BACILLUS CEREUS cuanacrenirics CHARACTERISTICS + aeroble, gram-poslive Ike rods + aerobic, gram-postive spore-forming ro + nonmotle + motile ” aa + spore-forming + Medusa head morphology ‘odry "ground glass” surface and irregular edges with projections along lines of Inoculation HABITAT and TRANSMISSION habliatis soll * transmission by contact with infected animals or inhalation of spores from animal halr and wool (woolsorter's disease) PATHOGENESIS: Virulence Factors antiphagocytie capsule (poly-Daglutamate) Cee « calmadulin-dependent adenylate cyclase ‘Lethal factor» inhibits a signal transduction In cell (7) division Protective» mediates the entry of the other two antigen(PA) components into ell How Anthrax Attacks Yerwaeae eeees SPECTRUM OF DISEASE o direct epidermal contact with spores causes formation of ‘mallgnant pustule with subsequent eschar and central necrosis (020% mortality rate inhaled spores from animal: (woolsorter's disease) or from weaponized preparations (bioterrorism) prolonged latent period (2 mos) before rapid deterioration massively enlarged mediastinal lymph nod hemorrhage, meningeal symptoms (©100% mortality rate without immediate treatment pulmonary ‘ingestion of live spores leads to UGI ulceration, edema and sepsis (rapidly-progressive course) ‘omortalty rate approaches 100% ‘TREATMENT + cutaneous anthrax ‘oDOCIs ciprofloxacin + Inhalational/ gastrointestinal anthrax. ‘DOC Is ciprofloxacin or doxycycline with one or two ‘Additional antibiotics (rifampin, vancomycin, penicillin, ‘imipenem, clindamycin, clarithromycin) TOP ti Zi. ZCAPD PREP MICREBIOLOCY HANDOUT BY PACIFICO ERIC E, CALDERON, M.D. ee eae era ees a ote tence ceenoprer noes esarcoreet HABITAT and TRANSMISSION * spores on grains such as rice survive ‘pores germinate when isha Bacillus cereus | Food polsoning from reheated fried rice? r Be serious! Bacillus cereus PATHOGENESIS + Heat-Labile Enterotoxin ‘ocholera-like enterotoxin causes ADP-ribosylation, Increasing cAMP a + Heat Stable Enterotoxin costaphylococeal-like enterotoxin functiohs steaming and rapid frying warm for many hours (e.g. superantigen SPECTRUM OF DISEASE: Food Polson ~ Twarnicron | DiARANGAL Fone ienedtoed rea Went veges neon ne non 6 men, 3) ean. 9) esioms "| Wining noes: | clasinroom, abdominal cramps sbdominal camps Gurtonthowa) | e40(meen a) | a03¢(menn a4) | ceterotodn | Weotan Mestioble SPECTRUM OF DISEASE * Opbthalmitis ‘occur after traumatic penetrating eye injuries ofthe eye with a soll-contaminated object ‘ocomplete loss of light perception within 48 hours of injury ‘TREATMENT + food potsoning (emetic/diarrheal) ‘symptomatic treatment only * ophthalmitis ‘ovancomycin, clindamycin, ciprofloxacin or gentamicin Wl, CLOSTRIDIUM TETANT ‘CHARACTERISTICS * anaerobic, gram-positive, pore-forming rods * spore is at one end (terminal spore) so organism looks like 2 tennis racket HABITAT and TRANSMISSION ‘habitats soit + entry through traumatic implantation into tissues with low ‘oxygenation leads to spore termination, growth, & toxin production PATHOGENESIS + spores germinate under anaerobic conditions in the wound + tetanus toxin (tetanospasmin) ‘protease that cleaves proteins involved in the release of ‘glycine from Renshaw cells in spinal cord Tetanospasmin Sila of Acton | _ Mechanism ofAcion [Presyrapic | iia raease of GABA terminals | by cleavage of potas |e rop hen of syrapic vesce lease) | laryngospasm, ore ysohagia, OTR Newonvsaior [Bioda reuctaneniter —|Weanestor | junction ‘alate paras Page 18 0f 71 COGY HANDOUT BY PACIFICO ERIC E, eee PEE Forlani nero con ee AT eR TERN sown oes sn reo weno — Tetanus toxin on ————_—_—. | Wound infection, ‘uptake into nervous, ‘Further toxin efiecte cau 7 penton ‘nem ana tae "wieepreed hte oP = =e aes es a = No ampere ‘musa when, a ‘TREATS a rau | caso Boag — > ri mnt of primary wound CHARACTERISTICS. - + Metronidazole or penicillin # TIG (atthe wound site) "vaccination with teranus toxoid ‘given in childhood and every 10 years thereafter ‘TETANUS PROPHYLAXIS : VACCINATION HISTOR Wouno | -Uneertain eres doses >a dois Texel Trig ars) | Tee {(TeANA) reanay | TIG(ATS) NO YES onlyit (lean, minor | yes No | lastdose | No | tiven> 10 years NO a | Yesoniyin | |contaminated| ves | ves | tastaose | WO, + |~ given >s QS ‘A 71-year-old male is brought tothe emergenc}iraam by his caretaker and presents with difficulty breathing, muscle | Migidity in the face, neck, back and upperexttemities, and | profuse sweating. The intern notes large-wound on his head | | hear the back of his right ear whigh his caretaker had bandaged up. The caretaker explains that the wound was the | result of fai while walking ip his backyard. The intern | performs a quick physical exam and observes increased reflexes. The patient was Intubated to assist in his breathing and was given diazépam, metronidazole, and an immunoglobullg after the blood work came back Which ofthe | | following neurotransmitters is affected inthis patient? ! ‘A Serotonin ! | i | B. Glutamate © Glycine D. Dopamine | Which ofthe following isthe mechanism of action of the | tatenospasmin? ‘Inhibiting protein synthesis ia ADP-ibesylation of) | elongation factor 2 B, Overactivating adenylat: cyclase by ADP-ribosylation of the Gs alpha subunit €. Overactivating adenylate cyclase by disabling the Gi subuni D. Preventing release of GABA by cleaving of synaptobrevin 2 E, Preventing relea 5 ‘of acetycholine by cleavit TOPNOTCH MEDICAL BOARD PREP MICROBIOLOGY HANDOUT BY PACIFICO ERIC E, CALDERON, M.D. + anaeroble gram-positive, spopeforinngrods HABITAT and TRANSMISSION’ ‘abliati sall vos organism and botivs con tanamted in mproperty preservedfood, ‘allaline vpsyabjediuchas green bean rmuehrooms* craiyocsnesis + botulinum coxin (heatiable neurotoxin) blocks acetylcholine + "elease causing flaccid paralysis (descending pattern) + elght immunologic types of toxin types A,B, and & are most common in humans + Botox a commercial preparation of exotoxin A ‘ouses: wrinkle removal, torticollis :CTRUM OF DISEASE: TOXIGENIC Food-Borne Botulism eye symptoms * BOV, diplopis, ptosis, mydriasis cobulbar signs (4 Ds) * diplopia, dysphonis, dysarthria, dysphagia ‘anticholinergic effects * dry mouth, constipation, abdominal pain bilateral descending flaccid paralysis ‘orespiratory paralysis: # ‘Triad of Botulism —— 1. Symmetric descending laccid paralysis (with prominent | bulbar involvement) 1 2 Absence of fever 1 | 3.lotact sensorium * Infant Botulism (Floppy Baby Syndrome) ‘owhen babies ingest spores found in household dust or honey odue to absence of. ‘traumatic implantation and germination of spores atthe wound site ‘TREATMENT + adequate ventilatory support ¢ elimination ofthe organism from GIT judicious use of gastric lavage and metronidazole or penicillin + trivalent botulinum antitoxin (types 4, B, E) Page 19 of 71 For inqutrles, visit www.topnotchboardprep.com.ph or https://www.facebook.com/topnotchmedicalboerdprep/ eke PREVENTION + proper sterilization ofall canned and vacuum-packed foods + adequate cooking « discard bulging/swollen cans V. CLOSTRIDIUM PERFRINGENS, CHARACTERISTICS anaeroble, gram-positive, spore-forming rods + double hemolyats on blood agar rowth on egg-yolk agar ‘ononmotile but with rapidly spreading growth on culture media HABITAT and TRANSMISSION, ‘habitats goll nd human colon + myonecrosis results from contamination of wound with soil or feces + food polsoning is trensmitted by ingestion of contaminated food PATHOGENESIS * SaaSangcane oxin, a lecithinase that cleaves cell membranes Gan produced by anasroble metabalism + Food Poisoning ‘production of enterotoxin which acts as superantigen ‘SPECTRUM OF DISEASE: TOXIGENIC * GanGangrens ‘opain, edema and cellulitis with crepltation, ‘ohemolysis and jaundice are common 08: to 16:hour Incubation period ‘ocharacterized by watery diarrhea with erampe & little vomiting ‘oresolves in 24 hours ‘TREATMENT + wound debridement and penicillin for gas gangrene + supportive management for food polsoning PREVENTION * proper wound care * adequate cooking Vi CLOSTRIDIUM DIFFICILE ‘CHARACTERISTICS + anaerobic, gram-positive, spore-forming rods « exotoxin in stool detected by cytopathic effect (fi ‘which viral ells infect cells) on cultured cells on males + carried inthe colon oS caiealthegunerlpopuaion AS oun ahead aa + tansmied bythe ecahoral + hands of hospital personne! are HABITAT and TRANSMISSION enn intermediaries xe: CEG. For inquiries, visit www.topnetchboardprep.com.ph or httpsi//www.facebook.com/tepnotchmedicalboardprep/ PATHOGENESIS + antibloties suppress normal flora allowing C difctie to overgrow ‘clindamycin, 24 and 314 generation cephalospori lindane 2 wphalosporins, + exotoxins A and B inhibit GTPases, leading to apoptosis and death of enterocytes ‘opseudomembranes are the visual result + infection can precipitate flare-ups of. ‘SPECTRUM OF DISEASE: TOXIGENIC Pagudomembranova Colitis ‘ononbloody diarrhea associated with pseudomembranes (yellow-white plaques) on the colonic muccas otoxle megacolon can occur TREATMENT + causative anblotc shouldbe withdrawn 3,8 + oral or vancomycinasiould be given ‘nd tds raped 2) + surgery iftoxle megacolon develops * ‘ Pye a = ‘The occurrence of pseudomenptttyiue coli maybe | ansclated t which eae : = ‘A. Use of eindamycta fag a Qt Infection B Rlce-wacer eo] Shigella dyeensth. |B. Clomid eodpaizoning [5S arian eid ad red i |e pulmonary abscess. During his hospital stay thea ominal pats and bloody darehes, and has a | Wocbes End srum creatinine © 09 g/d On teaching {pundg7bu ara hare eoposblepatogen producers | Betas that inden te colonc mutoneceosng stn | Iigpolymerizaton that results ncelldeathand mucosal | :crosis, What Is the preferred treatment for this condition? ‘A Continue Clinéamycin 8. Metronidazole Penilling [ie reason | CLOSTRIDIUM_ | Clostridium ZETani = TETanic paralysis Clostridium BOTullnum = BOTulism from bad BOTtles of food | Clostridium PEREringens = PERForatesagangrenousleg | (gas gangrene) ' | Clostridium Difficile = Dlarrhe Spode Gram ale Character petinges Lage wa wn dubiowane —“towcar Ge anda ped os Upae egave mettre ponte le ‘ho har may be ey pos gute rene CM = Mat commen belt td ie ‘fede tection get : sun san gan ngune cas htspetraite ‘smenin spread we clry tan gram rep be: lage en an pn gn commen ute to cal amples : am prbpes spas oot ony en sweet epost od WA amin Ls agent lad tan C pring ‘pee sok eatin pos, C mmonam sd C upto "ne ad wee pee “ een! “Ta bac vith anal pores “io: owe ns a wth ei pores appt ur fo lect nei: ps west poate Indu tie snd tae nope mpm Wie mig os ong amen Gct whe When tolaed rom tao ces ‘emia poe we ec, Indes «pale geod pate nso, id ene negate mara ie Wi clr wn atctve Ta ang bc wh pres seen (ceo prsdorentaous coi and “hon bande secre ‘ribeauodtad dues, ae) tamed: lon ound found eto ct hepa! ack ‘as appastig eles TOPMOTCH MEDICAL BOARD PREP MICROBIOLOGY HANDOUT BY PACIFICO ERIC E. CALDERON, M.D, Page 20 of 71 For Inquiries, visit www.topnotchboardprep.com.ph or https://www.facebook.com/tepnatchmedicalboardprep/ TOPNOTCH MEDICAL BOARD PREP MICROBIOLOGY HANDOUT BY PACIFICO ERIC E, CALDERON, M.D. For inquiries, visit www,topnotchboardprep.com.ph or https://www.facebook.com/topnotchmedicalboar dprep/ OTHER CLOSTRIDIA Vil. LISTERIA MONOCYTOGENES * Csepticum ‘CHARACTERISTICS ‘oNontraumatie myonecrosts in immunocompromised + aerobic, non-spore-forming, gram-positive rods patients «arranged in V- or shape + Csordelll . ‘motiliy ‘0 Toxle shock syndrome associated with septic abortion + Ceartium cold enhancement ‘Traumatic wound Infections ‘paradoxical growth In cold temperature Vi CORYNEBACTERIUM DIPATHERIAS HABITAT CHARA ics + colonizes Gland female GUT erable, non-spore-forming, non-motile gram-positive rods + widespread in animals, plants, and soll ‘lub or comma-shaped rods arranged in Vor L shape + looks like chinese characters ‘TRANSMISSION ‘matachromatie granules black colonies on cellurie plate «+ toxigenlety detected usin HABITAT and TRANSMISSION ‘ hableat is human throat «+ transmission via resplratory droplets PATHOGENESIS sxotoxin Inhibits protein synthesis by adding ADP-ribose to longation factor-2 (6F-2) subunit A, has ADP-rbosylating activity ‘subunit B, binds the toxin to cell surface exotoxin encodad by Brprophage smambranes result from death of mucosal epithelia! BUN | DIPHTHERIA | ABCDEEG of ipheneria | “ADP-ribosylation Beta-prophage i Corynebacterium Diphtheriae Elongation Factor-2 Granules (metachromatic) _ _ ‘BETA-PROPHAGE ABCDE of beta-prophage-encoded toxins ‘ShigA-like toxin (EHEC) Botulinum toxin Cholera toxin Diphtheria toxin ____Exythrogenic toxin (S. pyogenes). | 4 i = SPECTRUM OF DISEASE + prominent thick, ray, pseudomemban-over tonsils and throat + complications ‘alrway obstruction omyocardiis ‘cranial nerve and/or musele paralysis ‘TREATMENT + antitoxin and penicilin G PREVENTION + toxoid vaccine, usually in combination with tetanus toxold and pertussis vaccine (D1aP) ‘TOPNOTCH MEDICAL BOARD PREP MICROBIOLOGY HANDOUT BY PACIFICO ERIC E. CALDERON, M.D. + cross placenta or by contact during delivery * Ingestion of unpasteurized milk products, e.g. cheese PATHOGENESIS + Internati ‘interacts with Ecadherinon the surface * actin rockets “tet ‘propels the bacteria through tigymémbrane of one human calland ntoanother 5 SPECTRUM OF DISEASE : ec sommes on char al miscarrage or birth complicated by sepsis, gilgoFfan abscesses, and disseminated granulomas ofragsinjged during childbirth and manifests as meningitis «SE Manfigoencephalis ‘ajulcdstertosis ">, gbacteremil, sepsis, or meningitis in pregnant, elderly, or wo" fmmunocompromised individuals “TREATMENT + ampicillin with or without gentamicin PREVENTION + pregnant women and immunocompromised patients should ‘not Ingest unpasteurized milk products or raw vegetable Page 21 of 71 For inquirtes, visit www.topnotchboardprep.com.ph or https://www.facebook.com/topnotchmedicalboardprep/ TOPNOTCH MEDICAL BOARD PREP MICROBIOLOGY HANDOUT BY PACYFICO ERIC. CALDERON For inquires, vise wow. topnotchboardprep.com.ph or Rip www. facebook convtapneethneicalboardprep/ ALGORITHM - GRAM-NEGATIVE BACTERIA C GRAM-NEGATIVE COCCI _] SPECTRUM OP DISEASE T__ Neisseria meningitidis » Meningigis 1 Welsseria gonorrhoeae ‘emosgcomnion cause among aged 2-18.yrs flde e onhet naitrericnree ‘ofevgr headache, sf neck and an increased level of PMN .- spinal uid OVERVIEW ~ GRAM-NEGATIVE coccI 5 Meuliiiocnccamia *,,olssemination of meningococc! into the bloodstream GRAMSTAIN | OTHER FEATURES ‘ORGANISM __ + "omuttlorgan disease, consumprive coagulopathy, petechlal — oe — or purpurte rash (purpura fulminans) neapsulated,ferments| Neisseria + ___sndrome ‘omost severe form of meningococcemia nsec and guene'"| meng eam negtve heey hock werent prpur emia sea imassarcoguaiontrembesyopeni and adrenl lata caps, | neon inser een eee *Slne hemorrhage detracton ofthe adrenal t ance EEE ser ! beeen ae | pening (no sgican resttance) | tng at hs | pnevenion i casera Gbnarboee "ved conan pear plytcharde of sans AG Ya i acest eoaarrings vacin is capsular polysaccharide of strains ,C, Y,and ae ‘ocoupled to a carrier protein (diphtheria toxoid) to enhance 1. NEISSERIA MENINGITIDIS. immunogenicity ASA ERD SS ga ee ist ham Ones Sigramnegatve"idne-bean* dpococc moe igepotcearde pee } Sea ec eae Whih ofthe llowing vias acs aovs adhaane of : ee Bacteria the micot membranes ofthe eps eee emcees . | Bemtourinay and gaswointstinal trace? HABITAT and TRANSMISSION | 2 Ramrtaene teeta ‘habitats the URT |G congue +» transmission via respiratory droplets |__D. lpaprotese Which capsular type of Neiseria meningitidis: cks ‘humans are the only natural hosts Immunogenicity, hence, not included Ia the vaccine? « high carrlage rate in, ormivories, camps ‘military reer | mu ["RR PATHOGENESIS [Be + antihagooytic polysaccharide capsule ce + endotoxin (UPS) Ry + IgA protease LE ws «+ complement deficiencies in the late-acting complement components (C6-C9) predispose toillness ‘cannot form membrane-attack complexes TOPNOTCH MEDICAL BOARD PREP MICROBIOLOGY HANDOUT BY PACIFICO ERIC E, CALDERON, M.D. Page 22 of 71 For inauiries, visit www.toonotchbosrdoreo.com.ch or httos://www farshank.cam/tonnntchmedicalhoardoran! an bear tick Ti, NEISSERIA GONORRHOEAB. ‘CHARACTERISTICS s+ gramnegative “kidney-bean" diploccect ‘insignificant capsule . 3 oxidase-positive colonies on + ferment glucose only HABITAT and TRANSMISSION « habitats the buman genital tract ¢ eransmission by sexual contact or during passage through birth cans ww ‘The modified Thayer-Martin medium contains which | antibiotics? "A. Vorlconszole, cefuroxime, natilmicin | BL Vancomycin, colistin, nystatin i eetuetes 2 Lee at agen wae Sen 1, Leglonetepraumophte oven = can cae ODS eee Sea omens [ona 2 apa sepa | woes eee feocec™ | cee ‘components (C6-C9) predispose to illness ‘ocannét form lsordet-Gengou agar, | Bordetella + usual co-infection with Chlamydia trachomatis fee negative | whooping cough pertussis es src o iit Lote eareerae ae ‘o purulent conjunctivitis in newborns: svar stan, charcoal pavomophile ‘urethritis and epididymitis in met ‘most common cause of urethritis ‘omost common cause of PID ‘complications: * sterility + ectopic pregnancy * chronic pelvic pain * dyspareunia —___ syndrome (Perihepatitisy: volin-string adhesions ‘SPECTRUM OF DISEASE: DISSEMINATED ‘Omost common cause in sexually activegitults arthritis, tenosynovitis, or pustuleginghé skin ‘TREATMENT — * ceftriaxone + doxysycline due to fPequent co-infection with Chlamydia trachomatis PREVENTION =, + barrier contraception feordoms) + treat sexual part srythromycin-ointment or silver nitrate to prevent ECU ‘A 26-year-old male presents to his primary eare physician | with complaints of burning with urination, penile discnarge, and intermittent fevers. A urethral smear shows gram |, negative diplococc! within white blood cells. The organism | grows well when cultured on Thayer-Martin agar. The patient For inquiries, visit www.topnatchboardprep. com. a »ph or hetpa://vww facebook.com/topnotchmedicaoarcpier’ {1 OTHER NEISSERIACEAE ‘+ Eikenella corrodens and Kingella kingae cave culture- negative subacute bactertal endocarditis in patients with pre~ existing heart disease WEIN HACEK Organisms | naghs-HAGEK ng lagim sa mga may sakitsa pus 1 ‘Hoemophilus aphrophilus j ‘Actinobacillus ectinomycetemcomitans | Cardiobecterium hominis | (GRAM-NEGATIVE RODS: RESPIRATORY TRACT 7 Haemophilus nfivenzae "Small gram-negative (coccobacillary) rods + requires factor X (hemin) and V (NAD) for growth (Chocolate agar) . around S. aureus colonies HABITAT and TRANSMISSION « habitat is upper respiratory tract «transmission via respiratory droplets, PATHOGENESIS. + (ype b (polyribitol phosphate) = 95% of invasive disease + IgA protease + affects children from 6 months to 1 year ‘decline in maternal IgG and immature immune system ‘SPECTRUM OF DISEASE « sinusitis, otitis medla, pneumonia © meningitis: + eplglottitis ‘omast common cause cherry red epiglottis o__ sign + COPD exacerbations ‘Compare epiglottis with Taryngotrach obronchitis: Etiology: Radiographic sign: Treatment: | Is prescribed a course of ceftriaxone and the infection resolves | without further complication One year later, the patient | recurns with the same infection, Which of the following best | explains this lack of lasting immunity? | A-Antigenic variation - |B Polysaccharide capsule | © Bruton's agammaglobulinemia | D. Lack of necessary vaccination | HiVinfection ‘TOPNOTCH MEDICAL BOARD PREP MICROBIOLOGY HANDOUT BY PACIFICO ERIC E. CALDERON, M.D. For inquiries, visit www.topnotchboardprep.com.ph or https://www.facebook lus Influence ‘HaEMOPhilus influenzae causes Eplglotes | Meningitis Otitis media \ | | Preumonia | Page 23 of 71 m/topnotchmedicalboardprep/ For inquiries, visit www.topnotchboardprep.com.ph or https://www.facebook.com/topnotchmedicalboardprep/ ‘TREATMENT Facultative Intracellular Bacte + ceftriaxone ‘Some Bugs May Live EacultativeLy, 1 | “salmonella | PREVENTION | Brucella | « HIB vaccine contalning the type b capsular polysaccharide |, Mycobacterium | ‘conjugated to diphtheria toxold” Listeria ‘ogiven between 2 and 16 months of age |, Francie Legionella TT BORDETELLA PERTUSSTS Lyi ‘CHARACTERISTICS = + small gram-negative rods PATHOGENESIS. + culture on, agar or Regan-Lowe + endotoxin is sole virulence factor charcoal medium ¢ organism replicates atracellularty ‘cell-mediated immunity is important HABITAT and TRANSMISSION olnhibits phagolysesome fusion ‘« habitat is upper respiratory tract * predisposing factors * transmission via respiratory droplets ‘oold age, smoking, high alcohol intake, immunosuppression PATHOGENESIS SPECTRUM OF DISEASE ‘a » flamentous hemagglutinin '* ATYPICAL PNEUMONIA re ‘omediates attachment ‘o pneumonia accompanied by confusion, nonbloody +» pertussis toxin : diarrhea, hyponatremia, protelnutja and hematuria “ocauses ADP-ribosylation and lymphocytosis + PONTIAC FEVER A +* false adenylate cyclase omild, flulike illness * ‘olnhibits phagocytic activity «+ tracheal cytotoxin ‘damages ciliated cells causes whooping ‘SPECTRUM OF DISEASE ‘© paroxysmal partern of hacking coughs, accompanied by ‘production of copious amounts of mucus. that end with an {inspiratory "whoop" NATURAL HISTORY ‘TREATMENT + azithromycin or erythromycin PREVENTION + reducing cigarette and aleohol consumption « eliminating aerosolgifrom water sources « high temperatured and hyperchlorination in hospital water supply GRAM NEGATIVE RODS - GIT AND GUT Escherichia coll it Salmonella WL Shigella IV. Vibrio V. Campylobacter jejunt Vi. Helicobacter pylori VIL. Klebsiella pneumoniae Vill. Proteus mirabilis 1K Pseudomonas aeruginosa X. Acinetobacter boumannit XL. Flavobactertum meningosepticum Xil. Achromobacter xylosoxidans aka Alcaligenes sylosoxidans XilL, Bacteroides fragilis OVERVIEW - GRAM-NEGATIVE RODS - GIT AND GUT CGRAMSTAIN | _OTHERFEATURES ‘ORGANISM. cose fermenter, ‘TREATMENT seen seer Sewrihleet | ‘ lactone fermenter, erythromycin lueatepostve, eseu_| Nidslellapneumonioe PREVENTION comma-haped, + acellular vaecine In combination wth diphtheria and tetanus Jromneanive netics’ | vibiocaaoe toxoids (DTaP) " ross pst —— [comma-shaped, [IL LEGIONELLA PNEUMOPHILA eusouropc, | american! ‘CHARACTERISTICS fcomma-chaped urea + poorly gram-negative rods postive font ‘visualize with = «+ facultative ntracalluar bacterla ‘GRAMSTAIN | _ OTHERFEATURES. ‘ORGANISM + culture on charcoal yeast extract agar note, xidase ‘Salmanato spp. increased amounts of iron and cysteine nezative, H2s producer + rapid urinary antigen test |non-motile,cxidase | shigetia spp. nomaciose- [negate H2Srnon HABITAT and TRANSMISSION romiacone (reducer « habitat is environmental water sources leamnegaive [swarming ouase «+ transmisslon via aerosol from the water source rods Ineestve, 25. Proteusmireits « person-to-person transmission does not occur oroducer urease ewdave postive, Has Preudomencs nan producer obligate ae erupinasa ‘TOPMOTCH MEDICAL BOARD PREP MICROBIOLOGY HANDOUT BY PACIFICO ERIC E, CALDERON, M.D. For inquiries, visit www.topnotchboardprep.com.ph or https: Page 24 of 71 /Ieww.facebook.com/topnotchmedicalboardprep/ a us | ———4 | Baby Barbs was born premature at 33 weeks age of gestation. | | She was placed inthe neonatal intensive care unit. She | developed jaundice seven days afer. She was lethargic with poor suck Her anterior fontanel was bulging. The resident-on- uty suspected that she had meningitis. CSF analysis revealed increased WEG, Increased prota, dacreaved glucose. A Gram strain ofthe CSP revealed pink to red rod-shaped bacteria, Which ofthe fllowing organisms was most likely Isolated from her CSF? A. Klebsiella pneumonice B Bschertehia coll © Neisseria maningttdte D, Bacteroides rogilis 1, ESCHERICHIA COLL ‘CHARACTERISTICS « facultative gram-negative rods * lactose-farmenting colonies on EMB or MacConkey ager ‘ grean sheen on EMB agar; mecollic sheen «TSI agar shows acid slant and acid butt with gas but no H:S + typing by and H antigens ARGUE cose Fermentors LaGrose Is KEB Grow in MacCor r Citrobacter olive j | Klebsiella | -Kscherichia coli | | “Enterobacter | HABITAT AND TRANSMISSION « habitat is human colon + colonizes the vagina and urethra # transmission ‘ascending infection in UTI oduring birth in neonatal meningitis cfecal-oral route in diarrhea PATHOGENESIS * pill + flagelium (H) + capsule (K) + endatoxin (0) « enterotoxins ‘oHST and HLT cause watery diarrhea ‘overotoxin (Shiga-like) causes bl. rorowvsaccnanine ‘diarthea (HUS) ti ‘entigen TOPNOTCH MEDICAL BOARD PREP MICROBIOLOGY HANDOUT BY PACIFICO ERIC E, CALDERON, M.D. For inquiries, visit www.topnetchbeardprep.com.ph or https://www.facebook.com/topnotchmedicalboardprep/ (PSN setae ‘Think MESSY SPECK! Enterobacteria drink ] Morgandlla Serratia COEEEe! | Escherichia Proteus Capsular antigen (K) Shigella ‘Enterobacter ‘Qantigen Salmonela Strobactar Bageller antigen (4) ‘Yersinia ‘Bebsiella Esrments glucose | ake Enterobacteriacese | SPECTRUM OF DISEASE ‘* URINARY TRACT INFECTION ‘omost common couse ofcommunity-acquired UTI ‘+ NEONATAL MENINGITIS (026 most commen cause | BeE-L | Baroup srepracocl (6. agalactiae) Escherichia coll Usterla monocytogenes INTESTINAL INFECTIONS ‘TREATMENT smplelin oF sulfonamides for YT + 3 cephalosporins for meningitis and sepsis + rehydration i effective in traveler's diarrhea PREVENTION « limiting urinary catheterization, switching IV lines promptly, drinking botled water ‘The pathotypes of Escherichia cal are be cassified according | | to thelr characterise virulence properties and different | mechanisms tat cause dseare | | Towhichserorpe does the pathorpe EHEC belong? | "Rav-year-old male suddenly develops severe abdominal cramping and bloody diarrhea. The patient reports consuming undercooked ground beef four days prior tothe onset ofthe | symptoms. Which of the following best describes the toxin- | mediated mechanism of this disease process? | ‘A. Depolymerization of actin laments in gastrointestinal | | ‘mucosal cells, leading to mucosal cell death B, Increased intracellular AMP in gastrointestinal mucosal | cells, resulting in decreased absorption and increased secretion in the digestive tract | Inhibition of elongation factor-2 (EF-2) resulting in| decreased protein synthesis in gastrointestinal mucosal calls D. Inhibition of the 60S ribosomal subunit resulting in| decreased protein synthesis in gastrointestinal mucosal Ls ge 25 of 71 For inauiries. vist www.topnotchboardprep.com.ph or https://www.focebook,conv/topnatchmedicalboardprep/ Flea pomorcy mtoscas nono pnerwlcnosioLocy HANDOUT BY PACIFICO ERIE. CALDERON,H., + EE inate tw tpratitoudprep.com, ph or tpn ho aebenkcoeae ake eaboreprapy TrSALMONELLA SPP. ‘CHARACTERISTICS + frcultative gram-negative rods m-lactose-fermenting . detects antibodies in patient's serum ‘ultured in XED (aylose lysine deoxycholate) medium HABITAT. * human colon only ($. yphi) anteric tract of humans and animals, eg, chickens and domestic livestock (. enteritidis} ‘TRANSMISSION + fecal-oral route PATHOGENESIS: + Enteracalitis (5 enterttidts/ S. typhimurium) ‘tnvasion of the epithelial and subeplthelial tissue ofthe small and large intestines olnfectous dose is high ‘gastrectomy or use of antacid lowers the infectious dose signiicancy + Typhold Fever (5. yphi) ‘odue to Vi (virulence) capsular antigen ‘organisms enter, multiply in Payer patches, and then ta raticulovendothelalsyatern ‘predilection for invasion ofthe gallbladder, which can asultin establishment of thé ehroniecarrier state + Saptlcamla (S.choleraesuis) ‘obacteremia results inthe ‘osteomyelitis, pneumonia, common sequelae ‘© commonly inpatients with, cancer SPECTRUM OF DISEASE ‘oincubation period of 12-48 hours ‘onausea and vomiting and then progresses to abdominal ~~ ain and nonbloody diarchea + uphold Fever . ‘oincubation period S to21 days. : Week [Presentation [eure Sours 1 [Reza ear mae, Ya se marae Tene aes odbc isang Ty 2 [enpton meron, (ite so ese one (feces sare (Past eo cries ate] be sol bone nar ‘fever but litte orno enterociltis and then proceed to focal symptoms associated with the affected organ ‘frequently bone, ung, or meninges ‘TREATMENT + Ceftriaxone, ciprofloxacin + Phillppines: amoxicilin, chloramphenicol, co-trimoxazole PREVENTION + public health measures, eg, sewage disposal, chlorination hhand washing, food safety, + vaccines for Spl ‘TOPNOTCH MEDICAL BOARD PREP MICROBIOLOGY HANDOUT BY PACIFICO ERIC E, CALDERON, M.D. For inquiries, visit www.topnotchhoardoren.com.nh ar htrne-//awus farahani ram Mnmnabrtimadies Wi SHIGRLLA SP (CHARACTERISTICS oduce no gas from the fermentation of glucose + do not produce Mss ‘+ nonmatile ‘have O antigens + cultured In XD medium HABITAT ‘human colon only ‘TRANSMISSION + feca-oral route rT Pood, Fingers, Feces, Flies PATHOGENESIS a » Shigella invade the distal eum and colon >, low Infectious dose (highly nfyetioui) local inflammation accompa ueration oShigelia, * * most common couse df Ratllary dysentery oShigalla___ * most severe | ofbacillary dysentery ‘+ most comma equae of epidemie dysentery + some produce an gitarotoxn (Shiga toxin) EPONYMS. a ., Shigella dysenteriae Group's Sele ene 2 Sha peas pel Shiga bats e Shigella dysenteriae type 2 - Schmitz bacillus crop Ferner bacay | iB” _ Shoe fener Hse sel bets coup Naweate C7 _ Shel boa Manchester bac 8 Sige wane! Duralbats ‘SPECTRUM OF DISEASE ‘incubation period: 1-4 days, ‘fever and abdominal cramps, followed by diarrhea intially ‘watery then bloody diarrhea frequently resolves in 2 or 3 days ‘TREATMENT + Muid and electrolyte replacement in severe eases, ciprofloxacin PREVENTION * public health measures, eg, sewage disposal, chlorination, hhand washing, food safety IW, VIBRIO SPP. ‘CHARACTERISTICS + comma-shaped gram-negative rods + motile + oxidase-positive ‘+ cultured on ¢hlosulfate clerate bile salts sucrose (TCBS) agar . motility HABITAT ‘human colon only (V, cholerae) + saltwater (V. parahaemolyticus and V. vulnifus) ‘TRANSMISSION + fecal-oral route (V. cholerae) + contaminated raw seafood (V. parahaemolyticus) «trauma to skin, especlaly in shellfish handlers, or by ingestion of raw shellih (V. vulnificus) ‘TOPNOTCH MEDICAL BOARD PREP Mi For inquiries, visit www.topnotchboardp" PATHOGENESIS Tincerotaxin (eholeragen) acts by ADP ribosylation {J Imucinase enhances attachment the intestinal mucosa 1 high infectious dose + pandemics caused by Vibrio cholerae 0: biatype BI Tor (cholera BI Tor) ‘SPECTRUM OF DISEASE, * cholera (V. cholerae) ‘owatury dlarthea in large volumes (rice-water stools) ‘Seompllcations ‘cardiac and renal fallure + non-gap acidosis * hypokalemia + Gastroenteritis (V. parahaemolyticus, V. vulnificus) ‘oganerallysollimlted with an explosive onaet of watery ‘snd nausea, vomiting, abdominal cramps. headache, and low-grade feve + Wound infections (V, parahaemolyticus, V. vulnificus) ‘Oassoclated with exposure to contaminated water TREATMENT ‘ofluld and electrolyte replacement * tetracycline oF azithromycin shortens duration ‘ominocycline plus fueroquinclone or cefotaxime PREVENTION * public health measures ‘conly Improvements in sanitation can lead to effective control ofthe disease + short-term Immunity using cholera vaccine ‘omay cause herd immunlty V CAMPYLOBACTER JEJUNI ‘CHARACTERISTICS * comma-shaped gram-negative rods ‘= microaerophille “+ positive oxidase and catalase rests + grows well a 42°C on Skirrow agar HABITAT and TRANSMISSION + habitat is human and animal feces (cate, chickens, and dogs) « transmission is by the fecal-oral route PATHOGENESIS ‘= most common cause of bacterial gastroeitteritis + pathogenesis is unclear ‘possibly enterotoxin-mediated diarthes invades mucosa of the colon but dies not penetrate; therefore, sepsis rarely occurs ‘produces Mstologle damage tothe sucoral surfaces of jejunum ‘SPECTRUM OF DISEASE * watery, foul-smelling diarrhea followed by bloody stools accompanied by fever and severe abdominal pain (usually ‘undercooked chicken) ‘+ may mimic ulcerative coltis + disease associations 2 gros * antigenic cross-reactivity between oligosaccharides in ‘bacterial capsule and glycosphingolipids on surface of neural tissues reactive artrls (Reltr syndrome) CM outer syrome triad of urethritis, uveltis, and arthritis | CANT PEE | ANT SIE | | CANTeLIMBA TREE | TOPNOTCH MEDICAL BOAR® PREP MICROBIOLOGY HANDOUT BY PACIFICO ERIC £, CALDERON, M.D. IcROBIOLOGY HANDOUT BY PACIFICO ERIC E. CALDERON, M.D. -€om.ph OF htt Hiwwrw facebook.com/topnotchmedicalboardprep/ ‘TREATMENT + symptomatic treatment only + erythromyein for severe disease PREVENTION « public health measures Vi, HELICOBACTER PYLORI ‘CHARACTERISTICS ‘+ curved gram-negative rods 5 pi ‘= Frleroaerophille IU rs | particular Kinds Have Urease, | Proteus mirabilis ‘Hiebsialla pneumoniae | Helicobacter pylori Ureoplasma urealyticum HABITAT and TRANSMISSION « habieat Is che numan stomach + transmission ie by Ingestion. PATHOGENESIS + damages goblet cells of thé astric mucosa + production of large amounts of antmonta from urea by the rganiam’s urease ‘Oammonia also neutralizes stoinach acid, allowing the organism to survive + detected using the following tests: ‘OFGD with blopry showing H. pylort ‘urease breath test, H. pylor! stool antigen to document cure ‘SPECTRUM OF DISEASE ‘orecurrent pain In the upper abdomen, frequently accompanied by bleeding into the gastrointestinal tract + Disease Associations: ‘ogastric carcinoma ‘MALT lymphomas Vil, KLRBSIELLA PNEUMONIAE ‘CHARACTERISTICS + facultative gram-negative rods with large polysaccharide capsule + extended spectrum beta-lactamase (ESBL) activity in drug: resistant strains + urease-positive HABITAT ‘upper respiratory and GIT ‘TRANSMISSION «+ aspiration or inhalation « ascending spread of fecal fora ‘SPECTRUM OF DISEASE ‘thick, Bloody spurum (~ ‘usually nosocomial ‘omost common cause In alcoholics + Urinary Tract infections ‘TREATMENT + culture: guided treatment (cephalosporins alone or with aminoglycosides) _—" sputum) Page 27 of 71 For inquiries, visit www.tapnetchboardprep.com.ph or https://www.facebook.com/topnetchmedicalboaraprep/ Pores: ‘Vill, PROTBUS MIRABILIS. CHARACTERISTICS + facultative gram-nogetive rods on-lactose-fermenting + urease-positive motility PATHOGENESIS «urease hydrolyzes the urea in urine to form ammonia oralses pH producing alkaline urine ‘encourages the formation of struvite stones, composed of ‘magnestum-ammonium-phosphate ‘SPECTRUM OF DISEASE ‘UTI associated with nephrolichiasis ‘eataghorn caleull form on renal calyces ‘TREATMENT ‘TMP-SMX or amplellin «© surgery for large stones + gram-negative rods obligate aerobe af + non-lactase-fermenting * oxidase-positive + pyocyanin (blue-green pigment) » sweet graperlike odor + grown on Cetrimide agar Proud _ | Preudomonas ABRuglnosa isan obligate AERObE ____} HABITAT AND TRANSMISSION * environmental water sources, eg, in hospital respirators and. humidifiers F «inhabits the skin, upper respiratory tract and colon of abqut’s, “: 10% of people _, + transmission via water aerosols, aspiration and fecal |”? contataination 5 PATHOGENESIS: ; + endotoxin a + exotoxin A aw ‘otissve necrosis and inactivates ER.2, opel secretion system agin ranser + elastase and proteases me + pyocyanin damages the cli and thucosal cells + verdogiobin rom hemoglabln breakdown SPECTRUM OF DISEASE coburn wound hot wb félicills * spifpools, whirlpools, or inadequately chlorinated swimming pools and hot tubs skin graftloss duet infection green nll syndrome ‘puncture wound asteomyeliis ‘pubic osteomyelitis in IV drug abusers + Karinfections ‘omost common cause of otis externa —__indiabetics . + Chronle suppurative ous media TOPNOTCH MEDICAL BOARD PREP MICROBIOLOGY HANDOUT BY PACIFICO ERIC E. CALDERON, M.0, ITCH MEDICAL BOARD PREP MICROBIOLOGY HANDOUT B Sea ea ee rer nO prcomh or heat em PACIFICO ERIC E. CALDERON, M.D. facebook.com/topnotchmedicalboardprep/ ‘oventilacor-acguired pneumonia ohighsrisk CAP * Immunocompromised + broad spectrum antibiotics * steroid therapy + scructural ung lesions ~ bronchiectasis = sti fibrosis + GastralntestinaL infections ‘opphiitis (necrotizing enterocolitis) ‘Shanghal fever (ald form of yphold) opericonttis in patients undergoing peritoneal dialysis Unlnary Tract Infections ‘03 most common eause of nosocomial UTI Sepsis ‘oecthyma gangrenosum * hemorrhagie lesions ‘febrile neutropenia ‘TREATMENT + combination of active ar resistance to multiple an ‘oantipreudor lin (ticarcillin,plperacilin) ‘penleilin pi quired because of ‘ei clones (eiprofioxacin) ples of sultable combinations yp CEPTAZIDIME + AMIKACIN 5, bo PIPERACILLIN + AMIKAGIN ‘°AZLOCILLIN + CIPROFLOXACIN + rifampicin is added for refractory cases PREVENTION «+ disinfection of water-related equipment ‘hand washing «prompt removal of catheters XACINETOBACTER BAUMANN ‘ Pleomorphic aerobic gram-negative bacilus commonly isolated {from the hospital environment and hospitalized patients + Water organism, preferentially colonizes aquatic environments. + Ina hospltal setting Acinetobacter commonly eolonizes irrigating solutions and intravenous solutions + Low virulence, but capable of causing infection ‘oRespiratory secretions and urine, represent colonization rather than infection + Occur usually in organ systems that have a high fuld content * Nosocomial meningitis may occur in colonized neurosurgical patients with external ventricular drainage tubes + Multiesistant aerobic gram-negative bacilus sensitive to relatively few antibiotles + Maltdrug-resistant Acinetobacter Is nota new or emerging phenomenon, but A baumannil has always been an organism Inherently resistant to multiple antblotics 1X1 FLAVOBACTERTUM MENINGOSEPTICUM ‘formerly Chryseobactorium meningasepticum now called Eilzabethkingia meningosepticum + Nonfermenting, nonmotil, oxidase-positive Gram-negative aerobic bacillus * Ubiquitous in the environment found In freskwater, saltwater + Recently termed Blizabechkingla meningosepticum (or meningoseptica) + Aubiquitous Gram-negative bacillus Page 28 of 71 For inquirles, visit www.topnotchboardprep.com.ph or https://www.facebook.com/topnatchmedicalboardprep/ For inquiries, visie www.topnotehi + Historically associated with neonatal meningitis and infections ‘in immunocompromised patients ‘o Neonatal Infections ofean occur as outbreak with ‘environmental contamination belog the source + Infections are not common but are clinically important because the organism is naturally resistant to multiple antibiotice ‘XI ACHROMOBACTER RYLOSOKIDANS AKA ALCALIGENES XYLOSOXIDANS * Causes opportunistic Infections in comprovnlsed patients ‘Cancer patients + Catalase- and oxidase-positive, motile, gram-negative rod that oxidizes xylose and glucos environment and may be confused with ecles. + Unlike Pseudomonas, Achromobacter has peritrichous Nagella merging, unusual Gram-negative infection In CF CRAG Es XAll. BACTBROIDES FRAGHLAS. CHARACTERISTI anaerobic, gram-negative rods * predominant anaerobe of human colon spreads to blood or peritoneum during bowel trauma, Perforation or surgery PATHOGENESIS * Infections commonly due to combinations of bacteria in synergistic pathogenicity + LPS with low endotoxle activity * capsule (antiphagocytic and anticompiement) + foul-smelling discharge ‘short-chain farty-aclé products SPECTRUM OF DISEASE * abdominal abscess + peritonitis ‘TREATMENT + metronidazole, clindamycin or cefoxitin + surgical drainsge of abscess L OTHER BACTERIA _ T. Zoonoses IL Mycobacteria IL Actinomycetes IV. Mycoplasma V. Spirochetes VL Chlarydia Vil. Miscellaneous bacteria 1. MYCOBACTERIA TT LE TOPNOTCH MEDICAL BOARD PREP MICROBIOLOGY HANDOUT BY PACIFICO ERIC E. CALDERON, M.D. rdprep.com.ph oF htps://www facebook, com/tapnotchmedicalboardprep/ 1. ZOONOSES ee ncn to | Rese eay eee | oe | FAS RIOT ABET ORS nae ‘BRUCELLOSIS (UNDULATING FEVER) + small grara-negative rods without a capsule + wranm! EAM suet cele _, Unpastaurized dairy produets give you Ungulatng fever FRANCISELLA TULARENSIS TULAREMIA ee « small gramenegativerods—«. * + reservolr: rabbits, deer, and'roBents # transmission thks (0g, Dermacentor), Ingestion . + eatment: streptpiiyei or gentamicin YERSINIA PESTIS™ BUBONI IMONIC and SEPTICEMIC PLAGUE . nt bacteria + small grqet negative rods with bipolar (safety pin) staining {rete wild rodents *ywauamlsson: fe bite inalation :buboes, cutaneous hemorrhage ‘reatment streptomycin and tetracycline D. PASTEURELLA MULTOCIDA BITES and OSTEOMYELITIS + short, encapsulated gram-negative rod that exhibits bipolar staining buttery colonies with musty odor due to Iadole production + reservoir: cats and dogs * transmission: animal bites «+ treatment: penieilin G Zoonoses Bruce the COW Prancis the RABBIT | Yeye the RA‘ Papa the CAT Mycobacterium Wom ace, A tborcooats 1 oproe hewn agen amen ont) Trea nso Stonewe Tiras memes me scot wow dee ay Nonubersious Myesbecurit ie wn atmo om Me sercvocoun arbi inact sansa 4 maroon Cae proiiens “Dasma ni sna Cana envoy > Toon scores ay BE evar Motne Dominguet TOPNOTCH MEDICAL BOARD PREP MICROBIOLOGY HANDOUT BY PACIFICO ERIC E. CALDERON, M.D. Fae tnnudrlae viel warns tnmnntehbnerdnran ram nk ne bttnests ra Page 29 of 71 Yoowe TOPNOTCH MEDICAL BOARD PREP MICROBIOLOGY HANDOUT BY PACIFICO ERIC E. CALD For inquiries, visit www.tonnetchboardprep.com.ph or https:/ ovkes orvtch vor facebook, com/tapnatchmedicalboardprep/ PATHOGENESIS 2 MYCOBACTERIUM TUBERCULOSIS. ‘CHARACTERISTICS —— serobic, acid-fast rods ‘odlehI-Neelsen (or Kinyoun) + high lipid content ‘Omyeolle acids and wax D * produces catalase and niacin + slow-growing on Léwenstein-Jensen medium + lwclferase assay for drug resistance HABITAT and TRANSMISSION ‘habitat is human lungs + transmission is via respiratory droplets produced by coughing PATHOGENESIS: VIRULENCE FACTORS + exported repetitive protein ‘oprevents phagosome-lysosomal fusion . «most important virulence factor ‘e prevents leukocyze migration + tuberculin surface protein clita delayed hypersensitivity PATHOGENESIS: LYPES OF LESIONS + Exudative Lesions ‘opus from acute inflammstory response + Granulomatous Lesions ‘central area of bby azone of epithelioid eolis cotubercle is a granuloma surrounded by Mbrous tsue that thas undergone central caseation giant cells surrounded [MEASURES OF EXPOSURE ‘Purified Protain Dorivate Skin Test ive tast (“seroconversion”) Indicates: urrent infection/active disease ast exposure but not necessarily active disease ‘= BCG vaccination ‘negative test indicates: ™ no infection * anergy > immunocompromised, malnutrition, steroids, ~ and sarcoldosis type IV hypersensitivity mediated MECHANISMS OF RESISTANCE «Isoniazid ‘omutations of catalase-peroxidase - reduce intracellular transformation to active form _ 7 «+ Streptomycin sae ‘omutation of genes encoding the 30s Subunit + Rifampin ‘mutations in DNA-dependent RNA jolymerase PREVENTION dropletprecautions + BCG vaccine may limit extent of disease «Isoniazid for 9 months for patients who show seroconversion but no clinical symptoms ATYPICAL MYCOBACTERIUM 1B. MYCOBACTERIUM AVIUM-INTRACELLULARE COMPLEX (MAL MAC) = cause pulmonary disease in Immunocompromised hosts (AIDS patlents with CD4 «50) MYCOBACTERIUM LEPRAE (CHARACTERISTICS + aerobic, acid-fast rods + can’t be cultured in vitro: mouse footpad or in zhe armadillo HABITAT and TRANSMISSION «+ reservoir: humans and armadilios « transmission: prolonged exposure to nasal secretions of patients with the lepromatous form TOPNOTCH MEDICAL BOARD PREP MICROBIOLOGY HANDOUT BY PACIFICO ERIC E. CALDERON, M.D. aes Tapromatour Leprosy umber Telane = | Manyleson Titus deruction Numberofale-ort baci Ukatnoed of transmitting aprany [cetemadiatedresponsetoM.taprae| Present eprom gan ts _Ltpromatous leprosy is LEthal. SPECTRUM OF DISEASE + Tubersuloid Lenrany ‘ohypoplgmentod plaques, thickened superfcta nerves, and ‘significant anesthesia + apramatousLanrosy ‘leoninefactes erythema nodosum leprosum >, * tender red nodules or himpep oth shins + Hignals acute flare-ups of ¥igease ‘TREATMENT and PREVENTIOW: s tubereulold leprosy ‘odapsone and rifampin + Iepromatous leprosy. ‘dapsone rifampli, and clofaaimine + erythema nodosum leprosum IILACTINOMYCETES: ‘ACACTINOMYCES ISRAELI sBeqINOMYCOsIS— “a, #inaerobe (normal oral flora) ‘+ setting: local trauma (broken jaw or dertal extraction) + PE: hard, nontender swelling with sinus tracts draining «Treatment pentelllin G drainage 1B. NOCARDIA ASTEROIDES NOCARDIOSIS: + aerobic, filamentous gram-positive rods with aerial hyphae + weakly acid-fast (Fite-Faraco stain) + transmission: Inhalation of particles from soil * mane as myeetma and angand bran sacestes orange colonies) + treatment: TMP-SMX + drainage -Antiblotics for Actinomycetes i ‘S-N-A-P } | Sutfa for Nocardia: for Actinomyces, use Penicilin, IV, MYCOPLASMA PNEUMONIAE (CHARACTERISTICS «smallest free-living organisms ‘not seen on Gram-stain ‘ono cell wall ‘only bacteria with cholesterol in cell membrane + cultured in Zaton medium HABITAT and TRANSMISSION * habitats human respiratory tract + transmission {s via respiratory droplets PATHOGENESIS + tothe receptor 2 protaln ‘attachment inhibition of ellary moBon and neerons + hydrogen peroxide ‘contributes to the damage to the respiratory tract cells + autoantibodies against red eels (cold agglutinins) and brain, Jung, and liver calls rage 30 0f 71 Feee ce ctat elrwe tnenntchhaardnren ram nh ne bine: hvu farahank rnmitannntchmediealbaardoreo! rong ‘SPECTRUM OF DISEASE ‘+ most common type of atypical pneumonia (walking pneumonia) * clinical ndings not compauible with chest x-ray ‘+ extrarpulmonary manifestations: ‘hemolysis, Stavens-johnson syndrome (inost common {Infectious cause), Raynaud, Gullain-Barre syndrome ‘TREATMENT + DOG: erythromyein or azithromycin * penicillins and cephalosporins are inactive because the ‘organism has no cell wall ‘ore chrout have dissipated a week age, her hacking cough | eras chest radiograph demons Inher eg | Aityaraitone ns defo maa ade | \ Se i that appears much worse than the symptoms | cing. A cold agglutination test conducted on her | , blood shows clumping together when placed at 4 degrees! 8. Which of the following medications would best treat | the agent causing her cough? | A-Vancomycin |B Isoniazid |G Ampiciliin LD. Aatthromyein ed V.SPIROCHETES ‘A. TREPONEMA PALLIDUM. SS TRBPONEMA PALLIDUM (CHARACTERISTICS —— + colled splrochete }ot $een on Gram-stain because too thin * cannot be cultured in vitro ‘lacks trcarboxylic acid cycle (Kreb cycle) * microaerophilie HABITAT and TRANSMISSION + habitats the human genital tract + transmission is by sexual contact and from mother to fetus across the placenta . ‘SPECTRUM OF DISEASE ‘owithin hours, enters lymphatics and muhtplies“%,. . ‘local nontenderuler(chamere) usually for i220 weeks ‘ o diferente from painful CHANGRGID die to H,ducreyl @condyloma fata, maculopapular ras malaise, anorexia, lymphadenopaghy occurs after? to 3 months ‘omany years after inoculation clinical spectrum co granulomas (gummas) oneurosyphitis, otabes dorsalis : pupil + dementia paralytica ‘cardiovascular (aorttls) * obliterative invasion of small blood vessels and vasa vasorum, causing endarterieis ‘osnuffles /saddle nose mulberry molars ‘oHutchinson cried ‘ Hutchinson teeth, deafness, keratitis, saber shins ‘orhagades (angle of mouth) cofigoumenakis sgn (unilateral enlargement of the ‘sternoclavicular portion ofthe clavicle > detachment) ‘oiutton's joints (synovitis) pulmonary hemorrhage TOPNOTCH MEDICAL BOARD PREP MICROBIOLOGY HANDOUT BY PACIFICO ERICE.CALDERON, M.D. Ene inane wale nen aa NOTCH SOARD PREP JAICROBIOLOGY HANDOUT BY PACIFICO ERIC E. CALOERON, M.D. For nqurten, vse wer topnotchboardprep-com ph or Mpa facebeok comonnae hie icalboardprep/ DETECTION « Primary Syphilis ‘omost Important dlagnostc tests dark-fleld microscopy + scanner ye stonioncerssre SFE ot Finer OS 7 | Wat Can pay | BRED rosy | Brstoarian Se tecte crt : ESieonte [_ Besar | Sr ogc (ition | Brn i meatus Tenn png ‘fitch tere xeon ‘influenza-like symptoms fow hours after rcéiving penieilin, due to lysis of erepe PREVENTION Pa et + use of condoms, anublotes ater easy serologl follow-up eye ‘Whats the bais for anuigafiet7K nontreponemal (eagle) | test? . i ‘A. Lpopolysagetivide. B Mycollc eit, | © Cowhearreardigupin-ietthin D. Capsularpolysaccharides Lee ‘Eaundtud suncponrent ics « S'wpakly staining, gram-negative spirochetes largest medically-mportant bacteria + stain well with aniline dyes (Glemsa or Wright stain) ‘cultured on BSK mediura HABITAT and TRANSMISSION ‘+ ANIMAL RESERVOIRS ‘owhitesfooted mouse ‘owhite-talled deer ‘« TRANSMISSION ‘bite from deer ticks (Kxodes scupularts, trodes pacificus) ‘SPECTRUM OF DISEASE: Lyme Disease ‘ Suage1: + Stage 2: myocarditis (AV Bloc), meningitis, Bel palsy +» Stage 3: autoimmune migratory polyarthritis (onfon skin lesions), acrodermatitis chrontea atrophicans (EQUI Disease | BAKEa Key LYME pie | BelPatsy | Arete i | Kardiacbiock | [__ Erythema chronicum migrans ‘TREATMENT * early localized or disseminated Lyme disease ‘doxycycline (D0C},amoxicilin, cefuroxime «late manifestations ©lntravenous penicilin or ceftriaxone PREVENTION using insecticides * applying insect repellents to clothing * wearing protective clothing that reduces exposure of skin to Insects Page 31 of 71 m ‘TOPNOTCH MEDICAL BOARD PREP MICROBIOLOGY HANDOUT BY PACIFICO ERIC E, CALDERON, M.D. FEE For inquiries, viele www topnothbosrdprep.com.ph oF Ritps:/wvrw fecebook.comftopnoethieesicalboardprep! \ Tiberospina TeRROGANS 5 VI. CHLAMYDIA SPP. Cilanacranisics ote ee A. CHLAMYDIA TRACHOMATIS. ‘+ hook at one or both pointes ‘CHARACTERISTICS =e Serene + obligate intracellulerbacera relaaws — ergy paras tat se ot ATP Rector agi emseateeahlohnsen-eris (Dani) Se ecismucecaae { rowntn ptoneimid cure HABITAT and TRANSMISSION * €ytoplasmic inctustons in Giemsa ‘Thabane wl ang dome annals ‘¢ transmission via animal urine Ybligate Intracellular Bacteria odog, livestock, and rat urine stay Inside cells when It is Really Cold uni seting wading in floodwaters | "Rkese ! Goma J PaTHocenesis * locos panatrat cae mucous membranes sn onus ‘through small cuts or abrasions: * Elementary Body ‘+ multiply rapidly and damage endothelium of small blood vessels ‘inactive, extracellular “* organisms found in blood and SP early in the disease and in ‘enters cells by endocytosis trie duing eater + Reculate Body * (mmune complex-mediated meningitis and glomerulonephritis. ‘ometabolically active, intracellular... ‘seen microscopically at. Derecriow * darkfield microscopy is Insensitive cult EMJH, oa €® ana plcorme eyseoecoinser 2 * gold standard is leptospire microscopic agglutination test Cpeo aT) SPECTRUM OF DISEASE ‘ranges rom 2020 days ve fever cil intense headache — Si eaitendemess oe — * due to rapid multiplication of leptospires in muscles with hign oxygen tension conjunctival suffusion ~~’ EAE Forms of cstamydia * due to damaged and leaky conjunctival vessels = 2 | Elementary body = Enfectious, Enters cll via Endocytosis painful and itehy but with minimal tearing | Reticulate body = Replicates in cell by hesion eee + immune Lentospiruric Phase ‘aseptic meninglis HABITAT and TRANSMISSION * CSF pleocytosis wth or without meningeal’ symptoms + habitat isthe human genital ractand eyes * colncides with appearance of antibody ters + wansmission pulmonary invelvement ‘osemual contact * snow-flake lesions in CXR : ‘* most common STD overall hepatic necrosis ‘oduring passage of through birth canal oglomerulonephrits ‘ohand-to-eye contact * due to immune-complex deposition PATHOGENESIS + Well Syndrome + balance thats often reached between host and parasite, ‘omost severe form of leptospirosis resulting in prolonged persistence of nection triad: + Infection persists inthe presence of high antibody titers orange cast skin (severe jaundice) most common catise of death is respiratory failure due to SPECTRUM OF DISEASE ‘massive pulmonary hemorrhage + Trachoma 06 trachomatis types A-C ‘TREATMENT chronic keratoconjunctiviie progressing to scarring and «+ mild leptospirosis blindness odonycyclne, ampicilin or amoxicilin oalberstadter-Prowazek Inclusions -Rovnd to oval + severe leptospirosis ‘ntoplasmic inclusion bodles near the nuclei of conjunctival penicillin G, ampleilin, ceftriaxone or cefotaxime epithelia cells in trachoma * Jarisch-Herzheimer reaction may develop within hours after starting therapy MGM trachoma | TRACHOMA = B.C PREVENTION | ape pes + doxycycline chemoprophylaxis | Africa + vaccination of domestic livestock and pets | Blindness + rat control | Chronic infection ‘TOPNOTCH MEDICAL BOARD PREP MICROBIOLOGY HANDOUT BY PACIFICO ERIC E. CALDERON, M.D, Page 32 of 71 For inquiries, visit www.topnatchboardprep.com.ph or https://www.facebook.com/topnatchmedicalboardprep/ a = Bae inaaivion vv pnotchboerdprep.com.oh ar hitra://fenene fare OM, MD. + For inquiries, visit www.topnotchboardprep.com.ph or https://www.fecebook.com/topnatchmedicalboardprep/ —————— Oo Genital Tract Infections D. YERSINIA ENTEROCOLITICA Se ercoaman rey ata ‘MESENTERIC ADENITIS (pseudoappendicitis) ‘ NGU in males; PID in femdles eee * usual coinfection with gonorrhea piers idamapnenng-tter pan ‘oblrh complications: + transmission: oro-ecal route * neonatal conjunctivitis ————_—_—___—- + neonatal pneumonia E, KLEBSIELLA GRANULOMATIS. ‘GRANULOMA INGUINALE (DONOVANOSIS) ‘associated with Reiter syndrome incapsulated, pleomorphic gram-negative bacillus solar densities (Donovan bodies) lock like closed safety pins ‘0G erachomatis types D-K ‘+ small painless papule ulcerates to form late-onset (2-4 weeks) with velvety surface ‘striking tachypnea, characteristic paroxysmal cough * pseudobuboe formation cough), absence of fever, and «+ treatment: azithromycin [caiteeecaat is ‘sosinophilia CA ‘@C trachomatis types Li-L3 FUNGAL STRUCTURE ‘qworbasie forms ‘papule or vesleular which ulcerates and leads to buddl suppurative inguinal lymphadenitis (buboes) Hated. rheet jrene ‘postive Pre tert Eee eas Capes ieeyet, claenteen * Intradermal injection of antigen oumacteaaate prt hyphae B. CHLAMYDIA PNEUMONIAE + ‘* ATYPICAL PNEUMONIA ‘RUNGALPATHOGENESIS .. %,._ ‘oslmilar to Mycoplasma pneumoniae * two yes ofhost response; grgilomatousor pyogenic ‘associated with ceherosclerosts (Cell Microbial, 2004 Boby6(2):117-27 ‘0 Sero-epidemiological studies showing that patients wit] cardiovascular dlcease have higher titres of antic ‘response + some can be detected ty using skin tests for delayed + reduced celiumedlagad'Ymmunity predisposes to disseminated preumonae abodes cepted ene sede reteorenedreinae o Bunion ef theoraniom inisesersae lesions, but not in adjacent normal tissue by 1 IN ions erataey ater . organism from lesions \guushrodms phylloiain | 36 foemonion St at ih Taos eotapmee F ‘Ccntaivowa psirrach HX | apergitan —_* Inhalation ofthe spores causes allergic: Saree gg (ened ee | + ingestion of contaminated peanuts and fipergis "grains causes liver cancer due to + PSITTACOSIS bird exposure (bird fancier's disease) “et * parrots, parakeets, macaws, cockatiels... =)” LABORATORY DIAGNOSIS OF FUNGI osudien onset preumoni with malaise ee anita core SARORATORY throat photophobia, and severe headache" © dissolves human cells, allowing visualization ‘TREATMENT ~ Clow pH Tahibies the groweh of bacteria = STD zithromycin > + DNA probes for early infection + conjunctivitis ~erthromycig, + teats for fungal antigens or antibodies to fungal antigens Tov “tonyerdine S pstacosts —“aslbromyein Me | Which stain binds to the complex carbohyerate cell wall of VII. MISCELLANEOUS BACTERIA t fungi and lights up a ae ‘blue white color? 7 NTONELLAHENSELAE A fece h CAT SCRATCH DISEASE | © Caleofluor white D. Silver, + gram-negativerod + normal oral ffora of cats + cau-scratch fever in immunocompetent individuals ANTIFUNGAL THERAPY + bacillary anglomatosis in immunocompromised « transmission: cat bite or scratch 1B. GARDNERELLA VAGINALIS BACTERIAL VAGINOSIS. — + facultative gram-variable roa + clue cells present «+ positive whiff test (Aishy odor) + malodorous vaginal discharge ‘© HAEMOPHILUS DUCREY! CHANCROWD varod «small gram-negative r0¢ 1 Elta om choclate agar with eme (0) « PE: painful genital ulcer more Page 33 of 71 [MEDICAL BOARD PREP MICROBIOLOGY HANDOUT BY PACIFICO ERIC E. CALDERON, M.D. Pes. ‘isle www topnatchbeardprep.com.ph or https://www. facebook. comn/tepnotchmedicalboar dprep/ fungal agents pertain toa novel. highly | smisynthete polypeptides that inhibit | Importent consctuencs ofthe fungel cell wall? ‘A. Amphotericin B. Echinocandins i {G Allylamines [De imidazole |_E Triazoles ("Which the primary mechanism of resistance of C. albicans | against azole antifungals? | NA: Mutations in the 14-alpha-sterol demethylase |B Mutations in mlcrorubule-associated proteins C. Mutations in fungal squalene epoxidase D. Mutations in HMG-CoA reductase me —— CLINICAL MYCOLOGY [CUTANEOUS MYCOSES "DERMATOPHYTOSES. ‘CHARACTERISTICS + infect only superficial keratinized structures + 3 important dermatophytes ‘olerasporum-canis ‘odTrichophyton tonsurans ‘oEpidermophyton floccosum « tranamisslon: direct contact, dogs and cats PATHOGENESIS ‘ ehronletnfections often located in the warm, humid areas of the body + inflamed elrcular border containing papules and vesicles surrounding a clear area of relatively normal skin «hypersensitivity causes ‘0 Dermatophytidis an Inflammatory reaction te dermarophytosis ‘ata cutaneous site distant from the primary infection. ‘SPECTRUM OF DISEASE * Tinea pedis (athlete's foot) ‘Tinea ungulum ‘+ Tinea corporis ‘Tinea erurs (Jock itch) ‘= Tinea manum ‘Tinea capitis + Tinea barbae DIAGNOSIS 40%6KOMs septate hyphae ‘* Sabouraud agar: hyphae and cor + green fluorescence on Wp « superficial skin infection of cosmetic importance + bypopigmented areas + degradation of lipids leads to production of cds, and eventual destruction of melanocytes + spraner deuereensrtOt pearance on 10% KOK «» weatment: miconazole. ‘Walch organism causes fungemia in premature infants on 1V lipid supplements? ‘A. Tr'chophyton sp |B. Malassezia furfur C Blastomyces dermatiidis i _B. Condlidaelbeans | TOPNOTCH MEDICAL BOARD PREP MICROBIOLOGY HANDOUT BY PACIFICO ERIC E. CALDERON, M.D. For inquiries, visit wwrw.topnotchboardprep.com.ph or https://www.facebook.com/topnotchmediealboardpre TOPNOTCH MEDICAL BOARD PREP MICROBIOLOGY HANDOUT BY PACIFICO ERIC E- CALDERON: I Feree ct cei: wurn-tepnetchboardprep.com.ph or hetps://www.facebook,com/tepnetchmadicalboardprep/ ‘© SPOROTURIXSCHENCKUT ‘SPOROTRICHOSIS. + dimorphe fungus that lives om vegetation * occurs most often In gardeners, especially those who prune roses « transmission: thornopricl, + treatmentsttraconazale OR potassium lodide (ECUBUUETIN sporotrichosts | Planta rove inthe POT. {Sporocrichosis may also be treated with POTassium lodide) YSTEMIC MYCOSES A COCO ES ES nm ‘CHARACTERISTICS ‘= dimorphle fungus. ‘omold insoll tissue «+ endemic in arid regions of the southwestern United States and Latin America. fe ‘TRANSMISSION. yin + inhalation of@rehrospores,. a PATHOGENESIS hy © ahogpores form gpa led with endorpore framers nae = oo. & @ 5 HISTOPLASMA CAPSULATUM ‘CHARACTERISTICS + dimorphic fungus + forms two types of asexual spores ° © typleal thick walls and fingerlike projections © Important In laboratory Identification ‘smaller, thin, smooth-walled spores: ‘olfinhaled, rensmit the infection EPIDEMIOLOGY and TRANSMISSION + endemic in Ohio and Mississippi River valleys « growsin soil contaminated with bird droppings (____J or bat guano «transmission by inhalation of airborne microcanidia PATHOGENESIS + inhaled microconidia develop into buddingyeast inside macrophages «spreads tolivenand spleen + dissemination In those who have defectiveGMb Rarer ‘Histoplasma Hides within macrophages. Page 34 0f 71 For inquiries, visit www ‘G BLASTOMYCES DERMATITIOIS. ‘GHaRACTERIsTics lmorphle fungus round yeastwith nag * Endemic in eastern Norch America, (Ohla, Misissipp!) ‘TRANSMISSION MEUM siestomycosis | Blasto Buds Broadly, + dimorphle fungus a * thick yeast with multiple buds in wheel configuration * rostrleted to Central and South America ‘TRANSMISSION EI Paracoccisiovomycosis | PARAcoceldio PARAsals with the captain's wheat allthe way Latin America, | aa a I, OPPORTUNISTIC MYCOSIS ‘CHARACTERISTICS + normal flora of URT, GIT, FCUT * may appear as oval yeast witha single bud or as psoudohyphae : + form germ tubes in serum and ehlamydosspores in culture ‘SPECTAUM OF DISEASE oral thrush © wulvovaginitis > curd-like discharge ‘intertrigo ‘oskiminfections ~ satellite lesions onychomycosis + IMMUNOCOMPROMISED ‘oesophagitis ‘subcutaneous nodules right-sided endocarditis A 43-year-old HIV postive female presents with signs and) symptoms concerning for a fungal infection. Hes currently | ‘not on antiretrovirals and Ms:CD4 count is 98, Which ef the following candidal infections could be seen inthis patient but | would be very rare nan immunocempetant host? | ' | ‘A Ora hush 2 Vagina, |e inners | Dozsophagits BE Endocardis 1B. CRYPTOCOCCUS NEOFORMANS. CHARACTERISTICS + Oval yeast with narrow-based bud surrounded by a wide polysaccharide capsule seen in, preparation * positive latex agglutination test (GALAS) TRANSMISSION rows abundantly In soll containingbled (espectallypigeon) droppings ‘+ transmission by inhalatiomof airborne yeastceils TOPNOTCH MEDICAL BOARD PREP MICROBIOLOGY HANDOUT BY PACIFICO ERIC E, CALDERON, M.D. lores: rorparg nepal soma rner menouicusey waar evenoreome e cAaLDenon, : ‘epnetshboarsorep.cmn.oh or Ripa aeeboak. raha calboardprep) ‘SPECTRUM OF DISEASE + meningitis ‘vencephalitle, AG * drugs of choice ‘© amphotericin B ard Queytosine + chemoprophylaxis ‘fluconazole. @ ‘CHARACTERISTICS * xistonly as molds. "septate hyphae thatform______ branch TRANSMISSION aan * widely distributed tn nature We. * Inhalation ofarborme conidia, oat af 7 a | Think at ] Acute Angles in Aspergilu. (Rete Antes Asperetn ‘PATHOGENESIS ye ae infections: wounds, burns;thedormes, external ear, sinuses + aspergiloms (2 in lung eaves . pi (ABPA) ‘asthmatic symprpigwith expectoration of brownish bronchial puge’ TREATMENT | + drug of colcesamphotericnBs RHIZOPUS ORYZAE AND MUCORSEP. ‘tiveonsycosis. ‘aprophyticmolds with nonseptate hyphae withOUT walls and branches at right angles + rhino-orbital-cerebral infection with eschar formation * patients with , burns, or leukemia | 450-yearold HIV-positive male presents tothe ER witha | | erday history of fever and hemopeyss.Chestradiogrph shows a fbrocavitary lesion in the ight middle lobe Biopsy of | | theaffcted area demonstrates septate hyphae ta branch at | acute angles, Which of the folowing the mostlikely causal | organism? ‘Candide albicans 3. Preumocysjerovect | 6 Aspergitsjumiganus D, Mucor sp, __ | A3i-yearold female undergolng teaxment fr leukemia is | found tohavea frontal lobe abscess accompanled by j | paranasalsweling Sheaddtonally complains ofheadache, | | facial pain, and nasal discharge. Biopsy ofthe infected tissue | | would most ely eval whe af re flowing? | "A Yeas with paeudahyphae |B busting yet with naarrow base | C Septate hyphae yok i | D. Irregular non-septate hyphae |: Spherles conetingendospores i ‘E PHEUMOGYSTISIROVEGI (FORMERLY CARIN) ‘CHARACTERISTICS + daddetacminats organism, + major surface glycoprotein undergoes pragrammeds “Fearrangements ° diagnosis by staining BALwashings toluidine ive ‘metnvenamine siver stain Page 35 of 71 For inquiries, visit www.topnotchboardprep.com.ph or https://www.facebook.com/topnotchmedicalboardprep/ 7 Foros PATHOGENESIS # tranamission occurs by * cysts in alveoll Induce an inflammatory response consisting Piasma cells ‘© frothy exudate.hat blocks oxygen exchange poccurs when Cbee20 ‘© most common AIDS-defining lliness «+ giffose interatital pneumonia with ground glass infiltrate. ‘tilaterally " + 100% mortality if untreated, ‘TREATMENT and PREVENTION 4 drug of choice: teunethoprimsulfamethoxazoles + Forsulallergye oatovaquone © pentamidine edaprone a BEVIN. OGY Ae AB Ata) Seructure UL Viral Genetics il, Laboratory Diagnosis WV. DNAvirus ‘ V.RNAvirus VIRAL STRUCTURE Ta alze fram 20-300 nim # allviruses havea protein coat (capsid) © compoced of repeating capsomers in ome viruses, covered with lipoprotein envelope * nuclele acid genome + capsid s__Qucieocopsid VIRAL SYMMETRY + symmetry: spherical (Icosahedral) ot helical ical viruses are enveloped icosahedral viruses can be enveloped or naked VIRAL PROTEINS + surface proteins ‘attachment to host cell * DNA or RNA polymerases, ptors tnteracion Déueehnuceocapld and envelope emasion grhoc defenses VIRAL ENVELOPE + lipid membrane derived from the host cell ‘acquired as the virus exits from the cel n a process called all enveloped viruses acquire their envelope from plasma ‘membrane EXCEPT. (from nuciear membrane) «enveloped viruses are less stable and more-easily inactivated AECL naked viruses NAKED CPR and PAPP smear | Calle Parvo eee ‘TOPNOTCH MEDICAL BOARD PREP MICROBIOLOGY HANDOUT BY PACIFICO ERIC E, CALDERON, WD, TOPNOTCH MEDICAL BOARD PREP MICROBIOLOGY HANDOUT BY PACIFICO ERIC E. CALDERON, M.D. For Inquiries, visie www.topnotchboardprep.com.ph or Rttps://wwrw.facebook,com/topnetchmadicalboardprep/ { ‘The insterstitial space between the nucleocapel j tnvalope scaled the | ae Tegvesene | 8 Pet | & Hema D. Procapsid _ VIRAL GENETICS ‘VIRAL GENOME + all viruses arehaplotd except retroviruses * contain either DNA or RNA, but nat both ‘genomes can be elther single-stranded or double-stranded ‘genomes of RNA viruses ean be elther posicive:polarity or negative-polarity + some RNA viruses have a segmented genome MEQUEIUILNL sexmentedcenome 1 B-O-A-R Bunyaviruses | Orthomyxoviruses (influenza) ‘Arenaviruse Reoviruses NEGATIVE-STRAND RNA VIRUSES ‘+ must transcribe negative strand to positive + virion brings its own RNA-dependent polymerase Negative-Strand RNA Vi “acer ‘Or Eall Beplication. naviruses , Binyaviruses Parampxoviruse sQrhamyroviruses ' N, joPoviruses [Nmhabéoviruses VIRAL GENETICS ‘+ mutations cun produce antigenic, drug-resistant or attenuated variants + genomic reassortment causes epidemics, ‘example: influenza viruses + complementation: one virus produces a protein that can be used by another virus © example: hepatitis D and hepatitis B + phenotyple mixing: two different viruses infect the same cell © ao os a Orem omen i me (a \ ck , , oer # \ 7 om a 82 a ® LUNCOATING OF AN ENVELOPED VIRUS ey nepntne For inquiries, visit www.topnotchboardprep.com.ph or https://www.facebook.com/topnotchmedicalboardprep/ © JOTCH MEDICAL BOARD PREP MICROBIOLOGY HANDOUT BY PACIFICO ERIC E. CALDERON, M.D. Jircesce: ZerpeTe RRA onan peetoeariptieseoe ene een vee tecebeok.comn/topneechinedicalboarprep/ QUTCOMES OF VIRAL INFECTION —___ DNA VIRUSES Gytopathie + visualor funcional change nnfeced tect fis Malignant + oncogenic viruses induce Wansformavion transformation sod dnresvaned growth | coms + Infected cells appear normal, but are | Sommental —* Soducinglarge numbers st roere tiruses ee ine eee VIRULENCE FACTORS + cytokine decoys ‘o bind cytokines and block thetr ability to interact with Teceptors on thelr intended targets * virokines| ‘© reduce the expression of antigen presenting cells and Inactivate complement ‘antigenic variants of surface proteins PERSISTENT VIRAL INFECTIONS Carrier.» produce virus for long periads of ume sate + 6an serve as a source of infection for others Latent ¢ not producing virus at the present but can be Infections reactivated at subsequent time Slow Virus long incubation period, often measured in Infections years LABORATORY DIAGNOSIS DEFINITIVE DIAGNOSIS: * complement fixation + hemagglutination inhibition neutralization + uorescent-anibody assay | faplioma + radioimmunoassay . + Bnzyme-Linked Immunosorbent Assay (ELISA) > GOAERALITIES ON DNA VIRUSES ap [AIONA viruses have doublee SEROLOGIC TESTS aM [ANA vrs have Pry + seroconversion a a © finding antibody in one who previously had none a epitome, f, “fimmfianadipercmnege sen tnemerone |S tar iogosecurredtghaion ee | epoca antibody may be due to an . JstoNaviesareconedrel | powvins peligyl DETECTION OF VIRAL ANTIGENS — li, + presence of viral proteins, commonly used agnosis stow veunesreptcatein tbe | panies - atop ‘example: p24 of HIV and HbSAg. * nucleus EXCEPT 4 * presence of vrel DNA or RNA Is the Jbl supndard in viral como ee NAKED DNA VIRUSES < — — Virus Mest | Cncoganie Vaccine | Which isthe dist vaccine BWn to prevent a human cancer? 2 Tranamiasion ! Any ss Parvoviruseis | | “*PIratory No No BHBV Ysa, teanplacertal _ | c HPV ~ respiratory 9 Yes [One simon | wares | 4 | Which ofthe Blowing are the mast common sites ofvial | Taman emul e | | infedion and disease? paollomavius | skncontact | 1A Skin | Be Oropharyax and respiratory tract Co Gastrointestinal act {bens TOPNOTCH MEDICAL BOARD PREP MICROBIOLOGY HANDOUT BY PACIFICO ERIC E. CALDERON, M.D. Page 37 of 71 For inquiries, visit www, topnotchboardprep.com.ph or https://www.facebook.com/topnotchmedicalboardprep/ _® sees PARVOVIVIRIDAE iRVOVIRUS B19 (FIFTH DISEASE) CHARACTERISTICS Talked virus with cotabedral aymroetry inglevstranded DNA genome one serotype ‘TRANSMISSION + resplratory droplets and transplacental ‘SPECTRUM OF DISEASE ‘xythema Infectlosum (Fifth Disease) obrightred cheek rash (slapped cheeks) with fever, coryza, and sore throat laste _crisis plastic anemia in children slekta call ‘anemia, thalassemia or spherocytosis (1s trimester: fetal death 0294 trimester: hydrops fetalls arthritis ‘ofmmune-complex arthritis of small joints ‘9 pancytopenia in immunodeficient patients 2, ADENOVIRIDAE ‘ADENOVIRUS ‘CHARACTERISTICS ‘naked viruses with double-stranded linear DNA and Acosnhedralnucieocspeld virus with ber ¢Lantigenictyper ‘TRANSMISSION + aerosol droplet, fecal-oral, direct contact SPECTRUM OF DISEASE ‘+ URT: pharyngitis, conjunctivitis, coryza + LRT: bronchitis, atypical pneumonia «GIT: acute gastroenferitis cur: to. + histopathology ‘© Cowdry type B intranuclear inclusions cystitis 3. PAPOVAVIRIDAE WOMAN PAPILLON VIRDE - cuanacrenisTICs 5, naked rues wth doublerande ug DNA and an Kone cena AAS «lest 100 ypes rransuisson YS . street contact on emily 4 PATHOGENESIS: ~": "if aquniggal an bide oration of eps vacuole tes) «pesto? ‘lms oftumorsupisr ges Sesinnbie pss Serionbis ae SPECTRUM OF DISEASE + HEV-Lt04 ‘skin and plantar warts + HEV-Gand1i o genital warts, co respiratory tract paplionas ‘o most common viral STD ‘carcinoma cf cervix, penis, and anus ‘TOPNOTCH MEDICAL BOARD PREP MICROBIOLOGY HANDOUT BY PACIFICO ERIC E, CALDERON, M.D. TOPNOTCH MEDICAL BOARD PREP MICROBIOLOGY HANDOUT 8Y PACIFICO ERIC E. CALDERON, M.D. For inquiries, visit wew,topnatehboardprep.com.ph of https://www.facebook.com/topnotchmediealboardprep/ ‘TREATMENT tal warts: podophyllin kin warts: Iquld nitrogen «plantar warts salicyle acid PREVENTION vaccine for HPV-6, 11, 16 and 18 B.JCPOLYOMA VIRUS ‘only causes alsease in immunocompromised hosts + causes progressive multifocal leukow in alent wih ADDS eet eukowncerhalopaty (at) ‘demyelinating disease that affects oligodendrocytes characterized by deficits In speech, coordination, and memory CBKPOLYOMA VIRUS * only causes disease in immunocompromised Rost, « causes hemorrhagic cystitis and nephropachy’ain patients with solid organ (kidney) and bone marrow Mysplants ENVELOPED DNA VIRUSES HERPESVIRIDAE oS Veus | primarylnecion Route Tanemission svt |romautis fecratins and valve erp gentata ee one, eee 1 pctalintcon viv |vusicaia [zener [ia espeatory secretions lnectoue . (Vinreepratory overations CY menoaueeoss [NOM and save _ [consents [Anympromalie intrauterine, Wansfuions, |monenuclecss shedding [sexvalcontact. secretions : 'Sexulor onsen luncerain ‘epost searcoms | eee iransplastavon ‘A. HERPES SIMPLEX VIRUSES ‘CHARACTERISTICS + enveloped virus with leosahedral nucleocapsid and linear double-stranded DNA ‘TRANSMISSION ‘ HSV-1: saliva or direct «+ HSV-2: sexual or transvaginal PATHOGENESIS + vesile filed with virus particles and cell debris s steoflatengy ony: genial ganglla onsv-2: Unig Cra panglia + multinucleated glant cells are seen on Tzanck smear ‘large, pink to purple intranuclear inclusions (Cowdry pe A) SPECTRUM OF DISEASE ‘ogingivostomatitis coherpes labialis (Ups) ‘9 keratoconjunctviis ‘temporal lobe encephalltis ‘herpetic whitlow (fingers) ‘herpes gladiatorum (Crunk) genial herpes * palnfl anogenital vesicles coneonatal herpes * contact within birth canal aseptic meningitis Page 38 of 71 For inquiries, visit www,topnotehboardprep.com.ph or https'//www.facebook.com/topnatchmedicalboardprep/ ‘TREATMENT, + 0c is — Nal ‘oshortens tie duration of the lesions ‘0 reduces the extent of shedding of the virus Ono effecton the latent state BL VARIGELLA-ZOSTER VIRUS ‘CHARACTERISTICS + Enveloped virus with \cosakedral nucleocapsid and linear double-stranded DNA ‘TRANSMISSION . + respiratory droplets and by direct cofitact with lesions PATHOGENESIS Infeca URT, than spreads via the blood ro the skin + becomes latent inthe dorsal root gungla, which may reactivate aszoster + HP: muldnuclented glant cells with ntranuclear inclusions SPECTRUM OF DISEASE ‘Varicella olncubation period of 14-23 days. vesicular centrifugal rash *= *dewdrop on a rose petal” appearance co-complications: * pneumonia * encephgli:is oer Heye's © painful vesicles along de fo debilitating pain (pastherpetie neuralgia) Olnvolvement of geniculate ganglion causes facial paralysis ‘TREATMENT + mild: no treatment + moderate to severe: Acyclovir ‘ shortens the duration of te lesions ag T CYTOMEGALOVIRUS rm (CHARACTERISTICS T + Enveloped vas with osthedral aceocapedanieag- DNA a « cultured in shel! tubes . « + aegctive heterophil test rate ‘TRANSMISSION “ + human body lds, across the placettayirgaa raasplantation PATHOGENESIS 7 + immediate early proteins ‘translated from premade RNAs © impale assembly of fhe MHC cass -vral peptide complexes + HP: glant cells witfoyl's-eye nuclearinclusions ~ SPECTRUM OF DISEASE ‘most common cause of congenital abnormalities o milcrocephaly, seizures, deafess,Jaundee, and purpura ‘© most common when mother infected in 1* trimester ‘Heterophll-negative mononucleosis ‘fever lethargy, and abnormal lymphocytes in PBS ‘© pneumonitis, hepatitis, colitis, retinitis in Immunocompromised patients TREATMENT = docis_Gancyclevic | Fuscomet 1 conv is largely esstant to aycovir ‘TOPNOTCH MEDICAL BOARD PREP MICROBIOLOGY HANDOUT BY PACIFICO ERIC E. CALDERON, M.D. Ce ee eee opworen MINA. BOARD PREP NCROBOLOGY HANDOUT BY PACIICD ENCE. CALDERON, Wo, Plrerysise Tornoe eR one or ne pe inn facebook cam tpnotchmadk jealboardprep/ EPSTEIN-BARR VIRUS HAVA CHARACTERISTICS + enveloped virus with Icosahedral nucleocapsid and linear ds-DNA ‘TRANSMISSION + sali PATHOGENESIS ‘+ Infects mainly lymphoid cells, primarily B lymphocytes * eis EBV-specfic antibodies an nonspecific heterophit SPECTRUM OF DISEASE ovkising disease” iv sor hon ypbadeopay, npg mean ‘osplanlerupeurels a are complication.” * ald nero nie produce a enprg penc capsule ———— 1 | which age group? | neon os | | geese nee Fel | Bede | Sen aa ioppTa og tamcanconpronied pt | Seuund bitte ora Squamous epthelal cell with | seh owe | tet =) Ht | Malignancies ‘ooncogenicity associated with expression of latency sotated membrane proas 1.01 ote peti Aen pope oBealiymphonas cntisbagen! area ences poo cary lent AIDS paves HUMAN HERPESVIRUS: KAPOSI SARCOMA + malignancy of vascular endothelial ce!ls + shin esions dark purple fi to nodular and often appeat multiple sites « treatment: surgical excision radiation 2, POXVIRIDAE ‘X. VARIOLA VIRUS ‘SMALLPOX «= only disease that has been eradicated from the face of the Earth «+ largest viruses * brick-shaped poavirus containing linear dsNA + HP: Guarnler! bodies intracytoplasmic eosinophilic inclusions + transmission: aerosol or contact + incubation period of 7-14 days + prodrome of fever and malaise followed by centrifugal rash 'B. MOLLUSCUM CONTAGIOSUM VIRUS ‘MOLLUSCUM CONTAGIOSUM + pinkish, papular skin lesions with an umbilicated center «HP: Henderson-Peterson bodies ‘intracytoplasmic eosinophilic inclusions «+ eransmisslon: direct contact «+ treatment: Cifodavir Page 39 of 71 For Inquirles, visit ww.topnotchboardprep.com.ph or http://www. facebook,com/topnotchmedicalboardprep/ 5 3. HEPADNAVIRIDAE HEPATITIS B VIRUS ‘> Enveloped virus with Incomplete circular double-stranded DNA ‘TRANSMISSION « blood, during birth, sexual Hepatitides _ Hepatitis A» Asymptomatic atitis C= Chronic, Cirrhosts, Carcinoma, Carriers Heapucktonencmeninnecciaes {The VOWBLS (A and E) hit your BOWELS. ‘ eantigen (HBeAg) PATHOGENESIS ‘© only DNA virus that produces DNA by reverse transcription with mRNA asthe template ‘+ hepatocellular injury due to immune attack ‘OHBV has NO cytopathic effect PHASES OF DAMAGE + Proliferative ‘0 viral antigens expressed on’ hepatocyte surface host CDB+ T cells destroy infected hepatocytes * note that the virus dos not have a cytopathic effect + the host immune system (CD8+ T cells) does the damage! + Integrative ‘© HBV DNA Integrated into host genome (0 Fisk of HCC remains even after host mounts successful antibody response to virus SEROLOGICAL EVOLUTION Surfaceantigen * Describes whether the patient is diseased or immune + HBsAg ‘having this antigen means the patient has the disease (chronic, cute, or asymptomatie carrier) ‘o precedes onset of symptoms and elevation of liver enzymes + Anti HBsAg * ‘presence ofthis antibody indicates iatpatient is linrmune ‘and/or cured a ONO active dsease present “>, + Tells us how long the infection has been present HBeAg ‘othe antigen of the core ofthe virus (HBsAg removed) ‘antibodies are not protective but yield information about the state of infection ‘o positive antibodies Seen during the ‘window period” (a period of active infection) + IgM anti-HBcAg ‘onew infection is present (© most specifie marker for diagnosis of acute HBV infectior because it persists during the window period + IgG anti-HBcAg © od infection is present + the soluble component ofthe core antigen tells us how infective the patient is ‘01 soluble component of the viral core ° connotes high infectivity ore #1 “enfectivigy” + Ant-HBeAg ‘o presence connotes low infectivity ‘TOPNOTCH MEDICAL BOARD PREP MICROBIOLOGY HANDOUT BY PACIFICO ERIC E, CALDERON, M.D. TOPNOTCH MEDICAL BOARD PREP MICROBIOLOGY HANDOUT BY PACIFICO ERIC E. CALDERON, M.D. For inquiries, visle www.topnotchboardprep.com.ph or https://www.facebook.com/topnotchmedicalboardprep/ Negative ‘Postive | Negative | Fostive lg | Positive [Negative | Negative | PostiveigM [Negatiy Negative | Postive: Nugatve Pestve | Negative Negative Positive | Nagetive Negative | Poutve ‘SPECTRUM OF DISEASE « Incubation period: 10~32 weeks «fever, anorexia and jaundice ‘ dark urine, pale feces, and elewted transaminase levels + cirthosis and hepatocellular carcinoma carcinoma ‘oliver cell hyperplasia vieHBx protein that interferes with ps3 Bear rte xen P77 Ma ‘TREATMENT Ae + Incerteron-a and__lamivudine, __ PREVENTION ‘yiccination ‘first vaccine to prevent a human cancer [Reverse transcriptase functions as a/an | & Ruddependant RNA polymerase B. Topoisomerase |G DNA-dependent RNA polymerase [-_0._BlAcdependent DNA polymerase _ Lamivudine is an anti-retroviral drug which acts as an Inhibitor of reverse transcriptase. Hence, chis drug may be useful in the treatment of which ofthe following infections? ‘A. HIV and Hepatitis B B. HIV and Hepatitis D . Hepatitis B and Hepatitis D._Hepatitis B and Hepatitis D a RNA VIRUSES GENERALITIES ON RNA VIRUSES |All RNA viruses have single-stranded | Reovirus and |DNA EXCEPT Rotavirus All DNA viruses replicate in the Influenza and Jeytoplasm EXCEPT | Retrovirus Ld Page 40 of 71 For inquiries, visit www.topnotchboardprep.com.ph or https://www.facebook,com/topnatchmedicalbosrdprep! use TOR PNOTCH MEDICAL BOARD PREP MICROBIOLOGY HANDOUT BY PACIFICO ERIC E, CALDERON, M.D, For inquiries, visit www.topnetchboardprep.com.ph oF Rttpst//wew. fescebook.com/topnotehmedicalboardprep/ [can caas doen bu unocmaronned [ieee neuron ean fot [eereteonan i [ean deny eww iene Tame Te ‘CHARACTERISTICS qa + Naked nucieocapsld with singlnaip zed, postve:polarity RNA + classifeation based on pat in mice “TRANSMISSION Poe vorMeal spectrum oF piskasty, “MC epteg iron and ender veils In ooph °, oat and tender vesicles In oropharynx NAKED RNA VIRUSES } ew mouth disease TCORNAVIRIDAE 1 (esleybitgash on hands and feet and ulcerations in the mouth oe Conjunctivitis lprodynta Plcornaviridae o fo fever and severe pleurite type chest pain PERGHed ona BC (pea) |] [*\S Myocarditis and Pericardiis ollovirus _,| 2s" @mase common cause aaa ak ‘fever chest pln, and signs of congestive failure. | Conacte |“ *SEnsacl ws polovrus and echovi nus | ‘© Coxsackle virus, poliovirus and echovirus are | Hepatis A virus 1 ms aviridae ‘POLIOVIRUS POLIOMYELITIS and MENINGITIS « Naked nucleocapsid with single-stranded, positive-polariy RNA + oralfecal transmission 3. «replicates in motor neurons in anterior horn of spinal cord, couringparaysle + replicates in motor neupsigia anterior hor of spinal cord, causing paralysis + Hoserange is lmiel{a primates limitation is due tothe binding ofthe vifsbeapeid prin toa receptor found only on primate cel membranes. «+ Three serologic (antigenic) types based on different antigen determinants on the outer capsid proteins + Spectrum of disease (1) Inapparent asymptomatic infection © (2) abortive poliomyelitis ~ most common clinical form: mild febrile lIness with headache, sore throat, nausea, and vomiting 0 3) nonparalytic poliomyelitis - aseptic meningiie © (4) paralytic poliomyelitis - Naccid paralysis; permanent motor nerve damage + histopathology °. «+ prevented by vaccination Pehle (alk iPV) clive, attenuated (Sabin OPV) intranuclear inclusions TOPNOTCH MEDICAL BOARD PREP MICROBIOLOGY HANDOUT BY PACIFICO ERIC E. CALDERON, M.D. ‘© most common cause of aseptic meningitis CECHOVIRUS Enteric Gytopathic Human Qrphan + Called “orphans” because they were not initially associated with any disease * Now known to cause aseptic meningits, upper respiratory tract’ Infection, febrile illness with and without rash infantile diarrhea, and hemorrhagic conjunctivitis, «Transmitted by fecal-oral route. D. RHINOVIRUS ‘COMMON COLDS + Naked nucleocapsid viruses with single-stranded, posttive- polarity RNA ‘more than 100 serotypes + transmitted by aerosol droplets and hand-to-nose contact + Replicate better at 33°C than at37°C-> they affect primarily the nose and conjunctiva rather than the lower respiratory tract. + Acid-labile + killed by gastric acid when swallowed -> they do rot infect the gastrointestinal tract, unlike the enteroviruses. + Host range is limited to humans and chimpanzees REELINA wstnovirus RHINO has a runny nose Page 41 of 71 For inquires, visit www.topnotchboardprep.com.ph or https://www.facebook.com/topnotchmedicalboardprep/ F rornotcy >H MEDICAL BOARD PREP MICROBIOLOGY HANDOUT BY PACIFICO ERIC E. CALDERON, M.D. Flrcrrsise TOPMOTEH MEDICA toprotchboardprep.com.ph oF tps: /www facebook, com/opnetchmadicalboardprep/ E-HEPATIMIS A VIRUS HEPATITIS A + naked nucleocapsld virus with a ss-positiveRNA + fecal-oral transmission + children most frequently Infected * selfelirtted hepatitis * antl-HAV IgH is the most Important test ‘© Also known as Enterovires 72" + Has one serotype + There sno antigenic relationship to HBV or other hepatts ‘HEPATITISE ee # feeal-oral transmission ‘no chronic carrier state + no cirrhosis + no hepatocellular carcinoma «high morealty in, NORWALK VIRUS (NOROVIRUS) VIRAL GASTROENTERITIS: + Nonenveloped virus with icosahedral nucleocapsid und one piece of single-stranded, positive-polarity RNA. + Oral-fecal transmission ‘+ most common cause of nonbacterial diarrhe sudden onset of vomiting and diarrhea accompanied by fever LE ROTAVIRUS, ‘VIRAL GASTROENTERITIS — + Naked double-layered capsid with 10 or 11 segments of doublestranded RNA + most common cause of childhood diarrhea Rotavirus = | Bight Que The Anus |_ROTAvinus causes darrhea! a EI VIRUSES (NEGATIVE SENSE) INVELOPED RNA VIRUSES (NI E) | . 1, ORTHOMYXOVIRIDAE a TNFLUENZA VIRUS an ‘CHARACTERISTICS + Enveloped virus witha helical nuGlebcapsid and segmented, ss- negative RNA at + major antigens are hemggghitinn (H) and neuraminidase (N) + respiratory droplayransiission + Influenza A -> yoPtdijde epidemics (pandemics) ‘ Each year, igftadza is the most common cause of respiratobyjract infections. + Influenza*Be major outbreaks of influenza ‘Does natlead to pandemic + Influenza C -> mild resplratory tract infections but does not cause outbreaks of influenza. PATHOGENESIS + Envelope ~ two different types of spikes: HEMAGGLUTININ & [NEURAMINIDASE + influenza A virus ~has 16 ancgenically dstinet types of hemagglutinin (HA) and 9 antigenicaly distinct types of neuraminidase (NA). + Some of these types cause dlsease in humans. but most of the types typlcaly cause disease in other animal species such as ‘irds, horses, and pigs. TOPNOTCH MEDICAL BOARD PREP MICROBIOLOGY HANDOUT BY PACIFICO ERIC E, CALDERON, M.D. /Iwww.facebook.com/topnotchmedicalboardprep/ For inquirles, visit www.topnotchboardprep.com.ph cr htt HEMAGGLUTININ + Binds to the cell surface receptor (neuraminig seid alle acid) to initiate infection ofthe cell 1%, +The target ofneutralizing antibody NEURAMINIDASE + Cleaves neuraminte acid (si: virus from the ice 4 iso degrades the protedtive esplratory tract -> enhai ‘of mucus in the licas access to the respiratory Stale 7 + nay ca pau br hn vn safe Th are the source of the RNA segments that en ‘antigenic shift variants that cause epidemics among hurr | lan’and human influenza A virus infect the same cell .g. Ina farmer's respiratory tract) > reassortment > {Py ret vatat ofthe human A virus, earng the avian virus hemagglutinin may appear. « There is evidence that aquatic birds (waterfow!) are a common source of these new genes and that the reassortment event Teading to new human strains occurs in plgs. « Pigs serve as the "mixing bow!" within which the human, avian, and swine viruses reassort. ‘0 Waterfowl: H1 to H16,N1 to NS ‘o Humans: Hi, H2, and H3, Ni and N2 «Influenza B virus is only a human virus There is no animol source of new RNA segments. «Influenza B virus undergoes antigenic drift thatthe current strain must be included in the new version of the influenza ‘vaccine produced each year. ‘Influenza B virus kas no antigens in common with influenza A virus. Page 42 of 71 Seetereeenlilll EL - TOPNOTCH MEDICAL BOARD PREP MICROBIOLOGY HANDOUT BY PACIFICO ERIC E, CALDERON, M.D, Fer OI eee ae nn Aes een Rexbuakccetopnatiaateabaréprep! + ANTIGENE (pandemics) ‘© major changes based on the reassortment of segments of the genome RNA © EXAMPLE: when human flu A virus recombines with swine fu A virus ‘+ ANTIGENIC DRIFTS (epidemics) ‘on mutations in the genome RNA oe Antigenic dritt Testes prepara et o-e ¢-e escent en Ce toes 7 ‘passage of viruses from an infected call to uninfected cel et ae ‘Sra Mottons Kew irate comumaca ‘Stewand progrstve ‘Sudden chonge in the A MEASLES VIRUS change inthe molecular srvcture 9 ae ater comecstion coccinea ‘CHARA 3 microorganisms > ‘Rear lle or no «+ Enveloped virus with a helical nucleocapstd agdage Piece of ed ite Sequlred immunity 0 single-stranded, nagative-polarity RNA responses and thase NOVEL strains -> © Respiratory droplet transmission susceptibility, new epidemics or \ * = pandernes PATHOGENESIS ‘infect URT, then spreads to thelial cells «+ can transiently depress: immunity ‘Antigenic Shifts VS Drifts {HPs multinucleated glant bodtes) Sudden Shift is more deadly than graDual Drift. ‘agenda cune PANDEMIC. ' as ae Jao Sredls with a white, central dot on buccal mucosa aniigenieDrits cause EPIDEMICS, + pathognomo} cobright radi ~ extremities - palms/soles ‘number of persons have acquired mild symptoms of Infuenzs | pneumonia, subacute sclerosing espe beng vaccinated forthe appropratestainbeing | oC entncphalas GaP or Bmson dere | etre ou fad thatthe cltred tani he same ath | | tncorporated into the trivalent vaccine administered Cardinal Manfertations of Measles | throughout the wort You algo note thatthe strain had ahi 3 cs: Cough ] | ese tay ren PREVIOUS plan Cina very. ne Coryea new strains are slated and identified fr vaccine KS Conjunctivitis I | preparations. Whats the most likely explanation for thaw Koplik spots | GURRENT outoreak noted byte local heath depargpen? | 1a. Antigenic drift so | PREVENTION |B Antigenic si Co + infection confers flong tmmuniy | © Genomicreassortment . | vitamin reduces severity « LD. Complementation + prevented by giving live-attenuated vaccine raaaruner + DOC is OsetamivirorZanamivy, (™%, 5 MUMPS VIRUS + hosutalgrar nanan Sebo Yay CHARACTERISTICS ; dened tanta rarvern TO coe oN «toginay epee vera *iniegmesur ‘+ spreads to local lymph nodes and then via the bloodstream to Sn a oa a | reramor vase 8 “cn edt Parainfluenza + tender swelling of the parotid glands (parotitis) siecmtemete legs Sac rau aera ‘PaRaMyxoviruses 7 ++ complications: orchitis, meningitis | “Parainfluenza virus ! i Rsv WECM Mumps Measles virus. Mumps makes your parotids and testes as big as POM-poms. ies —— en rarvemon + infection confers lifelong immunity + prevented by giving Ive-attanuated vaccine ‘TOPNOTCH MEDICAL BOARD PREP MICROBIOLOGY HANDOUT BY PACIFICO ERIC E, CALDERON, M.D. Page 43 of 71 Fr inquiries, visit www.topnetchboardprep.com.ph or https://www.tacebook.com/opnotchmedicalboardprep/ TTT Te eee Heme = aaa inlets [RESPIRATORY SYNCITIAL VIRUS * Surtace spies are funon protelng, nov hemaggioUiTAF OF ‘Reuraminidases . ae For inquiries, visit www.topnotsnbesraprep-c TOPNOTCH MEDICAL BOARD PREP MICROBIOLO. 1GY HANDOUT BY PACIFICO ERIC E. CALDERON, M.D, -Om.ph or Nttps://www. etched /facebook.com/tapnotchmedicalboard WHO Guidelines for: [CategoryT Post-Exposure Prophylaxis ‘The fusion protein caus Blantoells(syncyui), + Humane a ells to fise, forming multinucleated hich give ree to che name ofthe virus the nararal hon GE RSV. VIRAL PNEUMONIA {MO important caus of pneumonia and bronehlosin infants : asain infants due to Immunologic crorsrasctia ‘with maternal antibodies eer een + treatment —— ‘B.PARAIWELUENZA ViRUS a AND 3 TARYNG IEOBRONCHITIS (CROUPY * characterized by inspiratory stridor, cough, and hoarseness > ———$——_____owny RHABDOVIRIDAE Ee “CHARACTERISTICS * Bullet-shaped enveloped virus with a helical nucleocupald and ‘one place of single-stranded, negative-polarlty RNA ‘TRANSMISSION ¢ animal reservoir: dogs, cats skunks, raccoons, and bats. ‘transmission by animal bice PATHOGENESIS. ‘+ multiplies locally at bite site, infects ‘moves by axonal transport to CNS. on bodies Virus 1oFy neurons, and ted tru skin ‘Virus replicates within strated muscles at site of inoculation Nervesare exposed tothe tirus at the neuromuscular and/or ‘neurotencinous spindles of unmyelinated sensory nerve endligs”* Virus spreads centrpetally up the nerve to the CNS and resi ‘the gray matior * Virus passes centritugally autonomic nerves tSkatndPtesuew CLINICAL MANIFESTATIONS + Prodromal period ‘9 Syinptom suggestive of ables: pareithess and/or fascleulatione at around the Bite + Encephaliti phase: excessive agttation ‘Periods of mental aberration gré interspersed with lucid intervals ca Prominent brainsterdysfunction + Coma/ death ‘pptor activity, excitation and DiAGNosis + Fluorescent antibody resting (direct and indirect) ‘0 Occurrence of rabies abs in the CSP is diagnostic for rabies since abs from vaccination do not crass the blood-brain barrier (© Abs in serum and CSF develop late in clinical course and ‘may be undetectable in the acute phase ‘+ RT-PCR on fresh saliva (viral shedding precedes signs) ‘* Skin biopsy sample (DFA or PCR) «Brain is optimal sample for defialtive post-mortem diagnosis & rabies is r/o in an animal only by DFA of brain tissue + Absence of Negri bodies does not rule out rabies. ‘TREATMENT AND PREVENTION + pre-exposure: vaccine + post-exposure: vaccine and immune globuli ‘o only vaccine that s routinely used post-exposure TOPMOTCH MEDICAL BOARD PREP MICROBIOLOGY HANDOUT BY For inquiries, visit www.topnotchboardprep.com.ph or https://www. “Touching or feeding ‘Noweatmont animals, licks on the sin ‘Catagory Il | Nibbling of uncovered sian, minor scratches or abrasions without bleeding. licks on broken sian Vaccine ‘Category TT | Single or multiple transdermal bites or scratches, contamination of smbrane with licks: exposure tobat bites or serstches Vaccine 7 ramunogiubutin ENVELOPED RNA VIRUSES (POSITIVE SENSE) 1, CORONAVIRIDAE Spaonavinys ‘CHARACTERISTICS « Enveloped virus with helical nuéfagcapsid and one piece of single-stranded, positive;polatity RNA + prominent club-shaped spikes form "corona + Bserorypes ‘TRANSMISSION + reservoir: horseshoe bat + Intermediate host: elvet cat « respiratpry droplet transmission SPECTRUM GF DISEASE + common colds "Severe Acute Resptratory syndrome (SARS) ‘o incubation period 2-10 days ‘atypical pneumonia rapidly progressing to ARDS. * virus binds to ACE-2 recepior © chest x-ray: non-cavitary "ground-glass”infltrates 2. FLAVIVIRIDAE —— ‘CHARACTERISTICS + Enveloped virus with icosahedral nucleocapsid and one piece of single-stranded, positive-polarity RNA ‘+ Flavivirus family © 4 serotypes ‘TRANSMISSION and DIAGNOSIS * transmitted by Aedes aegypti mosquito ‘diagnosed using dengue IgM * positive courmiquet test, Butis nonspecific ‘SPECTRUM OF DISEASE, + influenzalike syndrome with maculopartlar rash and severe pains in muscles and joints ) + labs: leukopenia, thrombocytopenia, increased hematocrit count * hemorrhagic shock due to cross-reacting antibody during 2~* dengue infection PREVENTION Insecticides * draining stagnant water + mosquito repellent 1B, HEPATITIS ¢ VIRUS ‘CHARACTERISTICS + Enveloped virus with one plecs of single-stranded, positive: polarity RNA + hypervariable region in envelope glycoprotein *# Gserotypes + diagnosis by anti-HCV or HCV RNA PACIFICO ERIC E, CALDERON, M.D, jebook,com/topnetchmedicalboardprep/ Pores “TRANSMISSION ‘most prevalent blood blood-borne ‘setting: 1V drug abusers + minor modes: needle-stick injury, vertical transmission, sexual For inguirt ne pathogen PATHOGENESIS « replication in liver enhanced by liver-specifle mlcro-RNA «hepatocellular inury due to immune attack ‘alcoholtsm greatly enhances rate of hepatocellular CA + ehronle carrlage of HCV ls much higher than HBV ‘SPECTRUM OF DISEASE * Incubation period 8 weeks * clinical presentation resembles hepatitis B * autolmmune reactions ‘ thyroiditis, autoantibodies, MPGN, PORPHYRIA CUTANEA, TARDA, ‘+ main cause of essential mlxed eryoglobulinemia ‘TREATMENT + acute hepatitis C: interferon * chronic Repaticis: pginterferon and ribavirin «liver transplantation for severe cirrhosis ‘omost common indication for. 3, TOGAVIRIDAE RUBELLAWI ‘CHARA ics « Envelopad virus with an eosahedral nucleocapsid and one ss- posltive-RNA + Transmission: respiratory droplet, transplacental SPECTRUM OF DISEASE incubation period 14-21 days ‘prodrome followed by 3-day maculopapuarrash and posterior auricular LAD face - trunk: arm/legs © immune-complex polyarthritis in adults infected uring the trimester Oatsocate abormalier patent ductus terns + Congenital cataract a + sensorineural deafness “N + eta reardation Congenital Rubin Sindrome 5 Bs of Congenital Rubella Syt 1e" | ees) ee | (cra ane) | (mental r¢ lon) (ner ear | Sy PnevenTion’ ‘infection ednfersfelong immunity { prevented by ping lve-etonuateé vaclne Stheuldnstbr pen to mmunccompromsed patents ot wepant wins 4. RETROVIRIDAE istingulshed from all other RNA viruses by the presence of an unusual enzyme, reverse transcriptase -> which converts a single-stranded RNA viral genome into double-stranded viral DNA. ‘TOPNOTCH MEDICAL BOARD PREP MICROBIOLOGY HANDOUT BY P; For Inauirles. visit www.toonotchboardoren.com.oh or httos://www.facebook.com/topnatchmedicalboardorep/ OPNOTCH MEDICAL BOARD PREP MICROBIOLOGY HAKDOUT BY PACIFICO ERIC E. CALDERON, M.D. re ,, visit www.tepnotchboardprup.com.ph or htty Jr! fncebook.com/topnotchmedicalboardprep/ he Reverseitygnscriptase \ CenTRA Doatygoag MOLECULAR GENETICS ome Ba", wa 99-9 prove Mt son Ration ~ HUMAN IMMUNODEFICIENCY VIRUS ‘CHARACTERISTICS + Enveloped virus with two copies (dipiotd) of a single-stranded, positive-polarity RNA genome ‘+ most complex ofthe known retroviruses + many serotypes STRUCTURE + Transmembrane proten, TM (fusion protein, also called gp41), Which linked toa surface protein, and SU (attachment proten,gp120) + Cone-shaped, icosahedral core containing the major capsid protein (CA also called p24) * MA (p17)~ directs entry of the double-stranded DNA provirus into the nucleus, andi ater essential forthe process of virus assembly. There are two Identical cops ofthe postive sense, single-stranded RNA genome in the capsid (that is, une other viruses, retroviruses are diploid). + The RNA s tightly complexed witha basic protein, NC(p7), ina nucleocapsid structure that differs in morphology among the Aiferentrewovirus genera, ar Soamaes) poeta ‘iosion Preomeae *ACIFICO ERIC E, CALDERON, M.D. Page 45 of 71 Bicrase Zornorer, meIcat s0aRd PREP MICROBIOLOGY HANDOUT BY PACIFICO ERIC E, CALDERON, Mm... TERY 7Sfingdinlas vate ww. topnotenbesroprep.com-ph or Retpsi//werw facebook. corm/topnscchmadicalbourdprep/ HIV Proviral Genome To designa vaccine agalnt HIV lnfeGon, logical goal would] | be alter some native molecule or product eranetrion in order to make highly immunogente Ifyou wish you prevent | the attachment ofthe virus to helper T iymphosyees whieh molecule or family of molacuies might best be targeted? apa | Bgpizo C gpa |__D. integrase ——_—-—_-! ‘TRANSMISSION Cc * original source: chimpanaéeay SBNOME:StructuralGenes 00 EN PROTEINS BNET + Sonica gi Bip) Nadeosnpid raneplacnal ‘ eedlntek la paryoopeegiiy I a Transcribes RNA genome + pref infects and kills helper ( many antigenic variants, ‘© CD4 500 to 200, lasts S years, mild mucocutaneous, + V3 loop - most immunogenic region of gp126 dermatologic and hematologic illnesses + Antibody neutralizes HIV infectivity, but the rapid appearance + PHASE 5- AIDS (© CD4 <200, lasts 2 years, AIDS-defining linesses develop of variants > difficult to prepare vaccine. ‘+ High mutation rate may be due to lack of an editing function in the reverse transcriptase. gp41 ~ mediates the fusion of the viral envelope with the cell membrane at the time of infection. H MEDICAL BOARD PREP MICROBIOLOGY HANDOUT BY PACIFICO ERIC E. CALDERON, M.D. Page 46 0f 71 aoe eo cen apnot nboar prep poof Rpt ww facebook com/Ropnothmedicboardrep/ OE Seep rw Aacebook.comitopnatchmedicalboardprep/ Meeperomenuuss ee — [____ MISCRLLANOUS VIRUS ‘cs |~ ETIOLOGY SERIE DROME HUMAN T-CELL LYMPHOTROPIC VIRUS (HTL [a tuberevosi | deseminated tabercuiovr ‘retovins cnsing ele Peal bolo ha — ae HSV esophagitis ‘associated myelopathy [¢. albicans [esophageal candidiasis ‘+ HP: malignant cells with lower-shaped nucleus ms pentane |p liroveer PCP pneumonia HBOLA VIRUS oe OL HEMORRTGTE PETER <200 IC. ins _|meningoencephalitis + "thread-like” viruses (C.Immitis coceldioidomycosis longest viruses ay * outbreak of he fever in Zaire (1976) parvum |ehroniedlarrhes + 100% morality rate M. aviurn invasive pulmonary disease <50 |H. dletum —_[histoy Osis JAPANESE B VIRUS v (CMV retin JAPANESE B ENCEPHALITIS 7 ‘+ member of flavivirus family iacnosis Premumptivedtagnots 9 deacon ofanibdisby LSA ‘© There are some false-poslive resuls with hs cert, * Definitive diagnosis > Westarn blot analysis ‘© Viral proteins are displayed by acrylamie gel ‘leerrophoress, ransferred to nltrocelilose paper (the blot) and reacted with the patient's serum. ‘lfantibodies are present, they will ind tothe viral proteins (pradominanely co the gp4i or p24 protein). © Bnsymatically labeled antibody to human IgG is then added A color reaction reveals the presence ofthe iIV antibody in the infected patient's cerum, * Grown in culture trom clsieal ‘Available only ata few medical centers, * Polymerase chain reaction (PCR) vary sensitive and specif: ©To detect HIV DNA within infécted call. © Some individuals who do nothave detectable antibodies have been shown by this testo be infected, ‘© Amount of viral RNA inthe plasma (Le, the viral load) can 0 be determined using PCR-based astays. TREATMENT * highly active antiretroviral therapy (HAART) Dtwo mulemie nibs advan and anu) ot protease inhibitor (ndinavi) i olmmune reconstton syndrome in patients co '* most common cause of epidemic encephall smecoreries sgt x» + temamited by Culer mosquitoes _ Infareta on CT scan 8 . SIOLOGICAL RELATION + Symblonis ‘Process of living unlike organisms + Lives inside the body of the host + Presence in host connotes INFECTION HOST RELATIONSHIP. 7 irre ney MAEINAL AS Obligate + Need ahostat some stage oftheir lifecycle | - I Parasite to complete development and propagation Oo | '¢ May exist in a free-living state but becomes Insp ib scion whe ot P| . sia feng ste | mast precive of te pee prog | Parate "parse when the ne | A. Level of HIV-1 in the plasma | B cereelcount | aepemmon © eecog el ao ae = Dy hate dele nat Sy | [fen aan absent Poraste rary 13 yearald wale wth coRS mutton | found abe immune 3. The patient's CRS Permanent perains on hortor fe | rutation interfer ‘function of which viral protein? ‘A. Reverse: 7 ‘Temporary ‘+ Lives on host for short period of time & iy, ! ‘+ Frees r sm that passes through Looe ! Seats torte ech hee | 7 [Aztyearcldmalewihatioryofnvinaeicawfomdo | sveesopwosrs have a CD4+ T lymphocyte count of 168 cells per microliter. As | Final | AconequncecFishVInedon ispatentsetnceased | {DENMVEGE IT nse maty ee ene Steet | ease at —o Rare te Semel ia neers oes j + Parasite doesnot develop further to | ae Host Later stages © salen tenatt | hoi yet ontnaa ad |B see fe a ae E bomen burrees (E5V) | tee svees or vectors ‘TOPNOTCH MEDICAL BOARD PREP MICROBIOLOGY HANDOUT BY PACIFICO ERIC E, CALDERON, M.D. + Vector: transl parasite fom one host» another * Blologi Vector: transmits parasite only aherithas completed its development + Mechanical or Phoretic Vector: Only transports the parasite Page 47 of 71 For inquiries, visit www. topnatchboardprep.com.ph or http://www. facebook, om/tapnotchmedlcalboardprep/ ‘SPECIAL TYPES OF INFECTION ‘infected Individual becomes his own direct source of infection ‘+ Hyperinfection or Superinfection ‘Infected Individual is further infected with same species leading to massive infection MODES OF TRANSMISSION + Contaminated food and water # Skin penetration * Arthropod vectors * Congenital ransmission =] = PROTOZOA T. Entameoba Nstalytiea 1. Giardia lemblia ML. Cryptosporidium paryum IV. Trichomonas vaginalis V. Plasmodium V1. Toxoplasma gondit Vil. Trypanosoma cruzi VIL. Trypanosoma brucei 1X Miscellaneous Protozoa os OS PROTOZOAN INFECTIONS . pical countries + Common in underdeveloped tropical ‘Poor tanita condos and hygighpctices © inadequate contral ofthe + inereased with world travel 3 Se * Euharyotes > have metabo prftses closer o those of the human hor han poke acti pathogent TReaTMenr oF pnptoZévinracrions + Protonoal dlseagag gees easly rested than bacterial infections + Many of gilyroou russe serous tone elec a the host ‘Calls showing high metabolic activity: neuronal, renal tubular, intestinal, and Bone marrow stem cells + Moschave not proved tbe safe for pregnant patients OVERVIEW - PROTOZOA an a ieee ae etnies See ee eee ee —— ton Ee — ee Iypanosome wp. Sleeping sickness ‘TOPNOTCH MEDICAL BOARD PREP MICROBIOLOGY HANDOUT BY PACIFICO ERIC E, CALDERON, M.D. For inquiries, visit www.topnotchboardprep.com.ph or https: romagice TOPNOTGH MEDICAL BOARD PREP MICROBIOLOGY HANDOUT BY PACIFICO ERIC E. CALDEROM, forse Ferrera soy my woromas oor wawwour ay paciico gus e canes [ENTAMOEBA HISTOLYTICA ‘PARASITE BIOLOGY + Peaudopodcforming nonflagellated protozoa -> Brownian movement + Most invasive parasite among Entamoeba + Eukaryotic organism that lacks membrane-bound organelles TRANSMISSION | INFECTIVE STAGE | DIAGNOSTIC STAGE] * wrophozoltes, + fecal-oral route | + mature cysts ‘mature oss, Immature cysts LIFE CYCLE OF ENTAMORBA HISTOLYTICA CDC Nrnemeot pte PATHOGENESIS: Virulence Factors «lectin mediates adherence ‘+ amebapores for penetration * ysteine proteases for cytopathic effect ‘SPECTRUM OF DISEASE + cyst carrier state + amebic colitis ‘dysentery without fever °. + ameboma assoclated with dysentery ‘+ amebic liver abscess ‘o most common extraintestinal form °. like aspirate colon ulcers ‘TREATMENT «= Tissue Amebicides ‘act on organisms in the bowel wall and the liver ‘EXAMPLES: chloroquine, emetines, metronidazole, tinidazole » Luminal Amebicides ‘actonly in the lumen of the bowel ‘© EXAMPLES: diloxanide furoate, lodoquinol, paromomycin Page 48 of 71 /iwww Sacabook.com/opnatchmedicalboardprep/ Ct JOPNOTCH MEDICAL BOARD PREP MICROBIOLOGY HANDOUT For inquiries, visie www.topnotchbe PACIFICO ERIC E. CALDERON, M.D, "aprep.com.ph or https://www.facebook.com/topnotchmedicalboardprep/ Taine Da ML. CRYPTOSPORIDIUM PARVUM bsveche oneeorin ‘PARASITE BIOLOGY + Opportunistic intestinal protozoa. Tail eran, + Undergoes schtzogony and gametogony * Autoinfection tn immunocompromised patients “TRANSMISSION [INFECTIVE STAGE | DIAGNOSHE STAG ESTAGE] * fecal-oral route | * thlekewalled | » thick-walled Mewoidzlpuuminal gan Mattnialo tial ps minal one? a ae ct Sarno aes ager . ‘limited non-bloody diarrhea ee EET Henle tance ——————— ose + covered with variant surface proteins TRANSMISSION INFECTIVE STAGE DIAGNOSTIC STAGE] ry ° + fecal-oral route = [Ls ressl-seel rota sig cysts = trophozoites, cysts __* trophozoites, cysts | LIFE CYCLE OF GIARDIA LAMBLIA 1V. TRICHOMONAS VAGINALIS. ‘PARASITE BIOLOGY + Urogenital protozoan + exists only asa trophozoite PATHOGENESIS >. \dhesive disc aritlectin facilitate attachment to avoid ([CTRANSMISSION [INFECTIVE STAGE] DIAGNOSTIC STAGE| i |» sexual, ean _ Fins op operand cont oe cytoskeleton + ing pone” ein ese, SPECTRUM OF DISEASE | . SPECTRUM OF DISEASE: Trichomon! “Ae annennt cere fen enaing ——-SECTROMOTDISSE. Tmo Me rowen eggs = companied by ltchng and burning + Chronic Infection 7 cervix ip consdpation, weight ss and ae « single oral dose of2 grams (four 500 mg tablets) af Tevoniazl Viatendne « Metronidazole 490171 MEDICAL BOARD PREP MICROBIOLOGY HANDOUT BY PACIFICO ERIC E. CALDERON, M.D. Page TOPNOTCH Men, visit www.topnotchboardprep.com.ph or https://www.facebook.com/topnetchmedicalbosrdprep/ For inquiries, f RR ee a yo ae Sera RE ee Ne Tie aeeeae oop tameardprep! LIFE CYCLE OF TRICHOMONAS VAGINALIS PATHOGENESIS + pathologic findings from the desruction of red blood cells CDC ‘release ofthe merozoltes and splenic eequestration of ei infected cells sSiecmea pe + people with RBC defect (GEPD, sickle cell) are immune to lara * partial immunity (___) seen in Individuals who completly recover from flelparum malaria LIFE CYCLE OF PLASMODIUM KEY LEARNING POINTS= Prulr of Pathology Hrawberry carve ‘eichomoniasis tongue 155, Kewasail SF albladder ——_cholesterclosis berry eee aneurysms ‘SAH, ADPKD, coarctation of aorta Afelclporum | Pvivax | malarice| ovale mulberry ‘hou | ahoon” | Thoun_ | ashowr rnolars congenial sys Rates| taner | aeier tumor tuberous sclerosis ; ‘ton | tesee__| quorum | twtan blueberry 2 erence ‘l rong RCs ‘muffin baby _.congenttal rubella, congenital CM a Ee arepelite ° Farstanin | Higa | tow | lowest | tow vesicles yasiiform mole Meron [0 | ue |e | a clusters S.qureus a Baran bunches sarcomabouyoldes Gameweyter | “htpeg | rtround | Compact | Sratound eet Reggie yA) Cecbaimais | We me] we | ne epiglontis eplglowttis “ay, er Recrodescence Ye No Yes No spotinmacula CAO, Tay-Sachs\lempan-Pick Talone Ca Pigment S marcescens, S rgrotnee | Many | few | few | few in CO potsoniegts, “+ a rarensanened coke SPECIAL CLINICAL OUTCOMES gametocytes 2 heparan . leftventricle hypertrophic cardiomyopathy ‘recurrence of symptoms after a temporary abatement (2-4 orange weeks) -peelskin “inflammatory breast CA coseen in P.feleiparum and P. molariae “astskin, “leptospirosis + RELAPSE colonies “ws Nasterldes ‘return of disease after its apparent cessation (1-6 mos) peor-shaped so duet reactivation of ypnozates ‘trophgzoltes Trichomonas, Giardia ‘seen in P, ovale and P. vivax = prostate gland DiAGwosis + Thin and ehiek smears with tersa stain thick smear to sereen forthe presence of organisms o thin smear for species idensifcation + Blood and tissue sporoaoa + Highest yield when blood samples taken during fever oF 2-3 ¢ Most important parasitic disease in man hours afer peak 1 Aserual lifecycle conse of shizogony and gametogony + Sexual life cyle involves sparogony MALARIAL DOTS + Schuffner Dots [TRANSMISSION | INFECTIVE STAGE | DIAGNOSTIC STAG: © puncate granultions presentin red bood cls invaded by + bie of infected a Povo and? vivo female mosquito | + sporotoltes feta + Maurer Dots h | f ‘coarse granulations present in ed blood cll invaded by P. fakiparom + Temann Dots ‘fine dots present in rad blood cells invaded by P. molaris ‘TOPNOTCH MEDICAL BOARD PREP MICROBIOLOGY HANDOUT BY PACIFICO ERIC E. CALDERON, M.D. Page 80 0f 71 jn /Ropnotchmedicalboardprep/ 1p MICROBIOLOGY HANDOUT BY PACIFICO ERIC E, CALDERON, M.D. othmedicatbourd ee gemma oistarat Dots PATHOGENESIS al «= two types of trophozoites ‘Schuffner dots = P, Ovale & Vivax ‘Rapidly multiplying tachyzoltes ‘SOVrang daming dots! = CMI limits spread Serena cduitashaped «P. lclparuM Se ————__—_ e eeporan i deve agnosis eee preered diagnostic art IgM angbody ¢ paroxysmal fever with malaise and bone pains LIFE CYCLE OF TOXOPLASMA GONDII «hemolytic anemia, jaundice and splenomegaly + parusitle pneumonitis ‘© cerebral malaria ‘o malarial oF Darck granulomas + acute renal fallure (blackwater fever) septic shock (alg/d malaria) ‘TREATMENT «* Tissue Schizonticides ‘kill schizonts in the liver © BXAMPLE: primaquine + Blood Schisonticides ‘kill these parasitic forms only inthe erythrocyte S EXAMPLES: chloroquine, quinine + cametocldes ‘hls gametocytes in human blood EXAMPLES: primaguine + sporonteider "prevent sporogony and multiplication inthe morguto © BXAMPLES:proguanl pyrimethamine (aay raion aa Srna nd? manana anes |rotnaiicen [Qian eT cence a slonaneient eon _| oR Cerrina + wrelnting raion | Savas comphenadileeron th | OR Mogving Male ence ease — om ames |g s Salta lt pyrnatiamine PROPHYLAXIS a ‘Drug | Prophylactic Use zl Vil, TRYPANOSOMA CRUZI "ChesseibeRrest without reliant [ipa PARASITE BIOLOGY Watsone areas wth ilaroqule-esant * Blood and vesue protozoan { Mefioguine_| Areas with chioroqulne-resstantP foleiparum =a + all4 forms: amastigote, promastigote, epimastigote, ‘Donyeyeline | Areas with multdrug-resistant?fakiporum ‘nypomastigote Soe ena pophyoir for od Pav CTO, + transmited by reduvld bug [rimacuine | pera forgery prevention J [TRANSMISSION _[ INPECTIVESTAGE | DIAGNOSTIC STAGE mo + reduvildbug Te meaeyclc | « uypomartgotes i {Tratomayine| " rypomastgotes_| inblood PATHOGENESIS ‘+ myocardial, glial, and reticuloendothelial cells are frequent sites + cardiac muscles the most frequently and severely affected tissue ehloroguine-resistantfaligglumtinalaria? ‘A. Atovaquone-progié B, Primaquine ©. Doxyeyeling, Ss {Bi Chloroquine *primaquine | Which agents useful rb treatiient and prophylaxis of | a DIAGNosIs sine d 8 or muscle Blopsy ‘Vi TOXOPLASMA GONDIT — Teale ofthe organism on special medium PARASITE BIOLOGY 5 « tissue protozoan ‘allowing an uninfected, laboratory-raised reduviid bug to ¢ definitive host isthe feed on the patient { humans and other mammals are intermediate hosts SPECTRUM OP DISEASE [CERANSWISSION —[ INFECTIVE STAGE | DIAGNOSTIC STAGE] + Asute Chagas’ disease * ingestion of cysts ‘© periorbital edema (___——____ gn) inraw meat « wopbosites o nodule near bite (chagoma) contaminated | + fecal oocyts " ‘fever, LAD and hepatosplenomegaly | food, aan + Chrunle disease [+ transplacentally ‘o myocarditis, megacolon, megaesophagus —__—_—) ‘TREATMENT + Nifurimox TTOPNOTCH MEDICAL BOARD PREP MICROBIOLOGY HANDOUT BY PACIFICO ERIC E, CALDERON, M.D. Page 51 of 71 For inquiries, visit www.topnotchboardprep.com.ph or https://www.facebook.com/topnotchmedicalboardprep/ TOPNOTCH MEDICAL BOARD PREP For inquiries, vishe wurn-topnocent LIFE CYCLE OF TRYPANOSOMA CRUZI ones 'dprep-com.ph ICROBIOLOGY Han «+ excessive somnolence + hyperesthesia ( sign) es :DOUT BY PACIFICO ERIC E. CALDEROM, 1.0, ‘oF htps:/ Awww. facebook.comopnetcimmedicalboardprep/ ‘Think WaGER! West = Gamblan Western Gate | East = Rhodesian Eastern Road | ‘SPECTRUM OF DISEASE ‘+ indurated skin ulcer (trypanosomal chancre) intermittent weekly fever and LAD. nlargement ofthe posterior cervical LN (Winterbottom sign) ncephalitis ‘plasma calls with cytoplasmic immunoglobul les Plasma cal with ytoplamisimmunoqlbuln global ‘TREATMENT {Siane Sage steer [irae ~~) I wetataan At Pennie [Sua eta ‘is ichanan | Elthnn | Wenioro ominneains aula i Sein [ Pete SE Laat anos | a a awa: cholea Tor American] ee ypanovomlasis? | ‘A. Inhibits trypanothione reductase | B. Crosses the blood-brain barrier | G Trypanocisl ‘against the trypomastigote form of 7. cruzi |_D. Only given as a nasal spray Vill, TRYPANOSOMA BRUCEI PARASITE BIOLOGY * Blood and tissue protozoan * Only 2 forms: epimastigote, trypomastigote * Remarkable. me # 2members 5 0 Trypanosoma brucei gambiense praitoa’ ‘© Trypanosoma brucel rhodesiense ‘TRANSMISSION « tsetse fly (Glossina) bite INFECTIVE STAGE ‘= metacyclic trypomastigotes DIAGNOSTIC STAGE * trypomastigotes in blood PATHOGENESIS “a «spread fom he sin hough thaplod tse lymph notes bran + somnolence (sleeping siéknass) progresses to coma due to demyelinating encephalltis(ARAS, brainstem) « cyclical fever spike very 2 weeks) due to antigente variation + Rhodesian more’rapid'and fatal than gambian LIFE CYCLE. OF TRYPANOSOMA BRUCEI eon tg ‘TOPNOTCH MEDICAL BOARD PREP MICROBIOLOGY HAKDOUT BY PACI ‘MISCELLANEOL IFICO ERIC E, CALDERON, M.D. aN A caNTH nT Granulgmatgut Ameoic tncephalts “tenance » nagetaferownsns APs Amebe Menlngoencephalits (fs Bireeving ameba + acquired while swimming in contaminated pools » BALANTIDIUM COLI Salantidial Dysentery + only. protozoan to cause human disease + associated with pigs + round-based, wide-necked Intestinal intestinal ulcers > BABESIA MICROTI Babestosis « transmitted by the bite ofthe frodes tick + Intraerythrocytic ring-shaped trophozoltes in tetrads in the form of a Maltese cross > LEISHMANIA SPP. Leishmaniasis + transtnitted by sandfly (Phlebotomus) + infective promastgotes » spectrum of disease ‘Cutaneous: Leishmania troplea ‘0 Visceral/kala-azar: Leishmania donovant ‘© Mucocutaneous: Lelshmanlo braziliensis > CYCLOSPORA CAVETANENSIS & ISOSPORA BELLI = coceidial sporozoa { cause diarrhea in immunocompromised patients Page 52 0f 71 tarahonk.comtoonotchmedicalboardprep! [ MS ena aE OO EE OP BED Oa ORE REISE. ca omnors mo NEMATODES: _] NAGNORIS ae NEMATODES cecal smear 1 Necator and Ancyeloscoma Katorkar technique WL Drehtrsttcture oe /.— Bnteroblus vermicularis y, Seorettar vio, ¢ major damage occurs daring larval migration Vi Capilaria phiipenonnt * principal sie of uasue reaction isthe lungs, where Vi Wecherenia andar \nflammation with an eosinophilic exudate oesies Vill. Trichinela spiralis * heavy worm burden may contribute to malnutrition 1% Miscellaneous Nematodes SPECTRUM OF DISEASE OVERVIEW - NEMATODES hypersensitivity peumonitis¢____ ation Dieesse ——Franeninion [restart {cute Intestinal obstruction AvcarasisTingestonoteggi [albendazole] * hepatobiliary ascariasis [aneiostoma larvalpenatraton ] {Pancreatitis | land Necator_|Heokworm = [rv Albendazole + malabsorption syndromes Trshurs |Whipworm [ingestonTeggt —|Maberdnncle"] * AUtvent deficiencies Intestines |énterobise |Pinworm —|ingestionotaggs [V"™! | TREATMENT CS. eamoute + Albendazole [Seonevior —larvalpenetraton } stones [sven ea Ivermectin as tun 1, NECATOR AND ANCYLOSTOMA S | copier |capitaisis Jlbendsicle | "PARASITE BIOLOGY Tatas hdercockad fh Soll-transmitted helminth the Flariass —|Mouqutobie — |oee qBlood-aucking nematodes ¢“*y Tissue ae + Hookworms fd Frtiate [sivas |eevriaéman [eben] ceetar americanus + Dilferentated basetho of buccal spearsof Marlferm 4 20-yaarold gir frem Forbes Park Makai has had abdominal] palin and cramps for the past few days, Her examination roduced normal Andinge except for nonspecific abdominal discomfort with a complete blood count showing anemia and TRAN INFECTIVE STAGE T DIAGN: + Alariformigrvae |» eggs in feces 224 eosinophils A stool specimen reveled the characters | 4g8 ofA lumbricldes. Which isthe most eppropriate o? peta 2 Albendazole CPaykecteea smear |b, Praziquantel Z ato-katz technique €. Piperazine | |. bition “s\"~" PATHOGENESIS ee Se/ «major damage due ta blood loss at ste of uachinent [Which rug used te eaten intl enaidag ‘© Blood consumed oozes in response to an anticoagulant | both ovicidal and larvicidal? Cc made by the worm | a Praziquantel ker | e anemia caused by blood loss |b Mebendazole ony te | & ledoquino! ne LIFE CYCLE OF NECATOR AND ANCYLOSTOMA d. Albendazole <— “ — | ASCARIS LUMBRICOIDES < PARASITE BIOLOGY qj y , — + most common and largest ied enatde * giant roundworm Ly * slanted helminth, [TRANSMISSION] INFECTIVE STAGE | DIAGNOSTICSTAGE + a * Saestion of Fexsintryonated egg | + unembryonated egg Ls. LIFE CYCLE GPUASCARIS LUMBRICOIDES Aememete beter aoe ‘SPECTRUM OF DISEASE Acute sate or ‘ground ttch atsite of entry Serpginous tacks (cutaneous larva migrans) © easinophiic pneumonia during transpulmonary passage mild GIT symptoms «Chronic disease ‘omicrocytic anemia cohypoalbuminemia jomieh Ousbin ‘TREATMENT aes + Albendazole 71 ICAL BOARD PREP MICROBIOLOGY HANDOUT BY PACIFICO ERIC E. CALDERON, M.D. Page 33 of Fer inquest www topnatchinordprep.com.9h 0 Rp /ewsraceSeok com topoehmevicaboaréprepr f + Jroersisy Tormoney MEDICAL BOARD PREP MICROBIOLOGY HANOOUT BY PAGIFICO ERIC E, CALDERON, M0, For inquiries, visit wwrw.topnotchbosruprep-com.ph or htipst, wow. facebook. com/opnetchmedicatboardprep/ i RICHURTS TRIERTORA LIFE CYCLE OF ENTEROBIUS VERMICULARIS PARASITE BIOLOGY Ageopien + whipworm aes cDC + sou-transmitted helminth @ _ [FRANSMISSION | INFECTIVE STAGE | DIAGNOSTIC STAGE] ‘Ingestion of eggs] + embryonated ogg | + unembryonated ei DIAGNosis * Direct fecal smear + Kato-katz technique: barrel-shaped eggs with bipolar plugs PATHOGENESIS ‘¢ burrow thelr hatrlike anterlor snds Into the Intestinal mucosa, LIFE CYCLE OF TRICHURIS TRICHIURA a> ON. 8 C V-STRONGYLOIDES STERCORALIS 1% * > ‘PARASITEBIOLOGY Se x + Soll-transmitted helminth Facultative parasites + ye.” -[RIAGNOSTIC STAGE) oN ‘SPECTRUM OF DISEASE * diarrhea The worms, ‘TREATMENT + Mebendazole 1V, ENTEROBIUS VERMICULARIS. ‘PARASITE BIOLOGY * Pinworm; seatworm . ‘+ soihtransmitted helminth ig + life cycle is confined to humans. ‘+ most common STH in deyeloped countries 30st lone "TRANSMISSION —[RMFEETIVE STAGE] DIAGNOSTIC STAGE] ibryonated | © eggs on perianal DIAGNOSIS eae olds . DiAGNosis® ..* PATHOGENESIS Graham technique + Larvae penetrate intestinal wall directly withcut leaving host D-shaped eggs and migrate to the lings (autoinfection) « eggs are rarely found in the stools © hyperinfection in immunocompromised patients PATHOGENESIS SPECTRUM OF DISEASE + female releases thousands of fertilized eggs on perianal skin + Acute disease ‘eggs develop into larvae, causing perianal pruritus, ‘oground lech at site of entry ‘an occur ‘omild watery dlarrhea eosinophilic pneumonia ‘SPECTRUM OF DISEASE: Enteroblasis * Chronic disease ‘+ pruritus ant oserpiginous track(______ + eosinophilic enterocolitis eduadenis + vulvovaginitis © paradoxical asthma pendicitis (oxyurtasts) co hyperinfection syndrome ‘TREATMENT ‘TREATMENT + Pyrantel pamoate + Ivermectin ‘TOPNOTCH MEDICAL BOARD PREP MICROBIOLOGY HANDOUT BY PACIFICO ERIC E. CALDERON, M.D. Page 34 of 71 Ene Inmaotae uiele unui Pnnnnerhinnarrincen ram nh ne brine: Hwan farahank com/tnnnatchmadicalboardoreo! FFlrormorcy ToPNoTcH MEDICAL BOARD PREP MICROBIOLOGY HANDOUT By PACIFICO ERIC E. CALDERON, M.D. FoF nquiriess visit wow topnotchbosrGprep.com,ph or https /wew.facebeok.cor/topnotchmedicalboardprep/ ‘VECAPILIARIAPHILIPPINENSIS__ PARASITE COMPARISON FaRAsiTE BIOLOGY (Characaretis | Wueherora gia intestinal nematode ‘Appenrance | Smoothly cured ct nly nematode whose lifecycle involves a migratory bird FFerminatnucel abate a Prevalence | Widespreed SEA only ‘Mosauto | Culex aadex |__vectors “Anopheles peed reference | Serotelivmphatics | _Umb hmhates Gina picture Elephantons Severity Les severe UFR CYCLE OF WUCHERERIA AND BRUGIA Manco rien — Qungarae ano oe oA DIAGNosIS: * Direct fecal smear + Kato-katz technique ‘© peanut-shaped eggs with flattened bipolar plugs PATHOGENESIS os +» embryonated eggs can cause autoinfection and hyperinfection, > ‘leads to vicerative and compressive degeneration of ‘enterocytes, resulting in sevgre malabsorption” cy ‘SPECTRUM OF DISEASE pee Acute disease 4%, ce borborygmus ~ Star bom pater * Chronic disease iN Sthrontiey dares. ¢ Soanma wang inpeesmmagibane ‘TREATMENT + Albendazole ‘Vil, WUCHERERIA AND BRUGIA PARASITE BIOLOGY * blood and tissue nematode ++ most debilitating nematode infection + 2 important causative agents 0 Wuchererta bancroftl Brugia malayi usual scenario © farmer fram abuca plantation "TRANSMISSION | INFECTIVE STAGE | DIAGNOSTIC STAGE| = mosquito bite * mlerofilarie TOPNOTCH MEDICAL BOARD PREP MICROBI For Inquiries, visit www.topnotchboardprep.com.ph or https://www. |OLOGY HANDOUT BY PACIFICO ERIC E. CALDERON, M.D. DIAGNOSIS * Thick blood smear © Curved or kinky microflariae + Specimen collection best done at night between 8 PM and 4 AM ‘o nocturnai periodicity + DEC provocation test PATHOGENESIS ‘+ adult worms in the lymph nodes cause inflammation that obstructs iymphatic vessels, causing (ymphedema + microflariae do not eause symptoms ‘SPECTRUM OF DISEASE + Acute disease ‘acute adenolymphangitis ofilarial fever ‘nocturnal wheezing (opleal pulmonary eosinophilia) ‘small eptheliolé granclomas (Meyers-Kouvenaar bodies) o expatriate sydrome «+ Chronic disease ‘ahydrocoele oelephantiasis o milky urine (chyluria) LOCAL EPIDEMIOLOGY + BANCROFTIAN FILARIASIS oe 0 Sorsogon, Samar, Leyte, Palawan, Camarines, Albay, Mindoro, Marinduque, Ronblon al of Mindanao + MALAYAN FILARIASIS Eastern Samar, Agusan del Sur, Palawan, Sulu ‘TREATMENT Page 35 of 71 facebook.com/topnotchmedicalboardprep/

Das könnte Ihnen auch gefallen